Permutation & Combination Spardha

You might also like

Download as pdf or txt
Download as pdf or txt
You are on page 1of 73

Chapter 6

Permutation & Combination

Contents : What and Where


6.1 Fundamental Principle of counting
In mathematics, a permutation of a set is,
(i) Multiplication Rule
loosely speaking, an arrangement of its
(ii) Addition Rule
members into a sequence or linear order, or if
6.2 Row Arrangement: Repeated objects
the set is already ordered, a rearrangement of
its elements. The word "permutation" also Number of Permutations with Repetition.
refers to the act or process of changing the 6.3 Combination
linear order of an ordered set. 6.4 Selection of one or more objects: (Total
Selections)
Permutations differ from combinations, which 6.5 Selection of Identical Things:
are selections of some members of a set 6.6 Selection and Arraignment
regardless of order. For example, written (A) Concept Equation
as tuples, there are six permutations of the set (B) Juggling with numbers.
{1, 2, 3}, namely (1, 2, 3), (1, 3, 2), (2, 1, 3), (C) Number of Functions:
(2, 3, 1), (3, 1, 2), and (3, 2, 1). These are all 6.7 Divisions and Distribution of distinct object
the possible orderings of this three-element Division into groups
set. Anagrams of words whose letters are Distribution
different are also permutations: the letters are 6.8 Divisions & Distribution: Identical objects
already ordered in the original word, and the Division into Groups
anagram is a reordering of the letters. The Distribution
study of permutations of finite sets is an 6.9 Multinomial Theorem
important topic in the fields Limitation
of combinatorics and group theory.
6.10 Circular Permutations
Permutations are used in almost every branch (1) Difference between clockwise and
of mathematics, and in many other fields of anticlockwise arrangement
science. In computer science, they are used for
(2) Theorem on circular permutations
analyzing sorting algorithms; in quantum
physics, for describing states of particles; and 6.11 Exponent of Prime p in n !
in biology, for describing RNA sequences.

2nd Floor, Opp. Domino’s, VIP Road, Zirakpur. Ph: 01762-351579, 9815301916, 9878170502
Chase your Dream Permutation and Combination Class : XI 2

Each of several possible ways in which a set or number of things can be ordered or arranged
“Life is full of permutations and combinations. Sometimes the order you do things matters sometimes it
doesn’t, but in order to find the solution in life you must work through each possibility present to find
your opportunity.”
Here we are to learn the most fascinating and thought provoking concept in mathematics. Surely it is going to
challenge you and without doubt even tease your mind in many ways.
Let us begin our journey will some of the problems which pop up in our mind, and we try to search for
sometimes in one breath.
1. India needs 8 runs to win in last three balls against Pakistan. Number of ways VIRAT and ROHIT will pull
it off.
2. No. of ways in which our annual examination having five subjects will be ordered.
3. Number of four digit numbers divisible by 4
4. Number of five letter words containing exactly there vowels.
5. Number of ways 5 toys can be distributed along 5 children.
6. Sum of all five digit numbers formed by 1, 2, 6, 7, 9 using once or more then once.
And innumerable other problems which involves cumbersome counting will be answered in this chapter.

6.1 Fundamental Principle of counting


(i) Multiplication Rule
If an operation (work) can be performed in ‘p’ different ways, and simultaneously another operation can
be performed in ‘q’ different mays them the both operations can be performed in p  q different ways.
Let say if a class consists of 30 boys and 20 girls and we have to select a boy and a girl monitor.
Now boy can be selected in 30 ways, if each boy is eligible to become a monitor, Similarly a girl can be
selected in 20 way’s. Now the both the operations (of selecting monitors) can be done in
30  20 = 600 ways
Please note this is a big number which if counted will consume our lot of time.
Also do understand that in these 600 ways if boy monitor ‘A’ is selected girl is changing in 20 ways.
So 20 pairs will be formed by fixing one boy monitor.
20 with another boy monitor. So finally, there are 600 pairs.

Record it, Memorize it, Note it.

If the concepts of AND is involved we apply product or multiplication rule.

Again if we have to select one shirt out of 5 available, AND one trouser out of 4 AND one shoe out of 6
available we can do it in
5  4  6 ways
If in your school you have 6 sections of class XI each having strength of 50, 50, 40, 40, 30, 20 and one
volunteers is to be selected from each section, then this can be done in
50  50  40  40  30  20 ways
This powerful rule will lead to solution of number of problems which are hard nut to crack by simply
counting.
Hold on we will soon but the cat out of the bag.

There was a 12 years old girl ‘ZOA’ one day during here leisure time ZOA started jumbling with the
letters of the ZOA. So she wrote all of them AOZ, AZO, OAZ, OZA, ZAO, ZOA.
Now, here is the Concept.
ZOA contains three letters
So the first place has 3 options, it can be filled with Z, O, or A AND the second place can be occupied
with 2 letters ( as one letter is a reach consumed ) AND third can be in one ways.

Note the concept of AND is involved.


So number of ways of filing three places can be done in
3  2  1 = 6 ways
Total 6 words are formed
Again if we want to arrange letters of and SIMRAN this can be done in
6  5  4  3  2  1 = 720 ways

2nd Floor, Opp. Domino’s, VIP Road, Zirakpur. Ph: 01762-351579, 9815301916, 9878170502
Chase your Dream Permutation and Combination Class : XI 3

Now you must be tempted to try with your own name you will your get the correct result, well if you are
lucky enough.
If your name is RONIT you will get correct result. But if it is ARYAN you will get the erroneous result.
But why ? Because ARYAN contain two similar letter so what is the solution, hang on we are almost
there, but first it is the time to consolidate our knowledge with few brain teasers.
Illustration
In a monthly test, the teacher decides that there will be three questions, one from each of exercise 7, 8
and 9 of the text book. If there are 12 questions in exercise 7, 18 in exercise 8 and 9 in exercise 9, in how
many ways can three questions be selected
(A) 1944 (B) 1499 (C) 4991 (D) None of these
Solution: (A)
There are 12 questions in exercise 7. So, one question from exercise 7 can be selected in 12 ways.
Exercise 8 contains 18 questions. So, second question can be selected in 18 ways. There are 9
questions in exercise 9. So, third question can be selected in 9 ways. Hence, three questions can be
selected in 12 × 18 × 9 = 1944 ways.

(ii) Addition Rule : If an operation can be performed in ‘p’ different ways OR in ‘q’ different ways
then the operation can be performed in p + q different ways.
Let say, a class consists of 50 students, 30 boys and 20 girls. If we went to select a monitor, it can be
boy or a girl monitor.
Boy monitor can be selected in 30 ways or a girl monitor can be selected in 20 ways.
So total ways  Boy OR Girl
30 + 20 = 50
If the concept of OR is involved then the we go for addition.
For reaching Delhi from Chandigarh we have 10 trains. 30 buses and 6 flights, so no ways will be
10 + 30 + 6 = 46
Train OR bus OR Flight
Illustration
A college offers 7 courses in the morning and 5 in the evening. The number of ways a student can select
exactly one course, either in the morning or in the evening
(A) 27 (B) 15 (C) 12 (D) 35
Solution: (C)
The student has seven choices from the morning courses out of which he can select one course in 7
ways.
For the evening course, he has 5 choices out of which he can select one course in 5 ways.
Hence he has total number of 7 + 5 = 12 choices

Row Arrangement : Distinct objects


The numbers of arrangement of ‘n’ distinct objects in a row is given by
n. (n - 1) (n - 2) - - - - - - - - 3.2.1 = n !
Concept : 1st place can be filled in n ways AND
2nd place can be filled in (n-1) ways AND
3rd place can be filled in (n -2) ways AND ……so on.
Now last place can be filled in one ways. As the concept of AND is involved we have multiplication rule.
Record it, Memorize it, Note it, this result is applicable only if these are distinct objects.
 Numbers of ways of arranging letters of the word JUMBLE are 6!
 Numbers of ways of arranging girls in a row is 7 !
 Numbers of ways of arranging letters of the word GANESH JI is 8 !

Now, Let us look beyond this concept.


If letter of the word GANESH JI are to be arranged and the word starts with a vowel, then numbers of ways
will be
3  7  6  5  4  3  2 1 = 3  7
1

As first place can be filled in three ways as we have only 3 vowels. Fixing one vowel at first play remaining
other letter can be arranged in 7! Ways.
 Again if vowels occupy only first three places.

2nd Floor, Opp. Domino’s, VIP Road, Zirakpur. Ph: 01762-351579, 9815301916, 9878170502
Chase your Dream Permutation and Combination Class : XI 4

A E I G N S H J

3! 5!
Now first three places can be filled in 3! ways and other five places with consonants can be filled in 5! ways.
So total number of ways are 3!  5! .
Note again multiplication rule is applied as we have to fill first three places with vowels AND last three places
with consonants simultaneously.
 Again, if vowels are together but not necessarily at first three places.
Now grouping vowels together and consonants as separate objects .
G A E I N S H J
So we have 6 distinct objects, which can be arranged in 6! ways
But at the same time three vowels can relatively exchange their position which can be done in 3! ways.
So number of ways are 3!  6!

Let play with another word EDUCATION


 Total number of arranging letters without any restriction is 9!
 Now if vowels are at odd places.
We have five odd places and 5 vowels. So vowels can be arranged in 5! ways and remaining four places
can be filled by consonants in 4! Ways
So total no ways 5!  4!
Note again we have multiplication rule as three is a concept of AND. We have to arrange vowels AND
consonants both operations (work) simultaneously.
 If the condition says that numbers of ways in which A comes before E.
Now this problem involves some brain work, Let’s analyse it:
In all the arrangements of letters of the word EDUCTION, A and E relatively change their position in 2!
ways
i.e, A E and E A
So if we divide the total arrangement i.e, 9! by 2! ,we get the number of ways in which A always comes
before E.
 Again if condition sys that A comes before E and E comes before O, then total arrangement can be
divided by 3! ways. Guess Why ?
….. A ….. E ……. O …..
In all the arrangements A, E and O relatively change their positions in 3 ! ways and we need only one
arrangement out of these 3! i.e, 6 arrangements, first A, second E then O.
9!
So, required number of way : 5 .
3!
Before jumping to another concept, let us come across with some brain teasers.
Illustration
Find the number of ways letters of the word MONDAY be arranged in row such that
(i) word starts with letters A
(ii) word starts with A and ends with O
(iii) words in which O and A are always together
Solution
(i) Fixing A at the first place,
A
Remaining 5 letters can be arranged in 5! = 120 ways.
(ii) Again fixing A and O at first and last place
A O
Remaining 4 letters can be arranged in 4! = 24 ways
(iii) Grouping A and O together, we have five different objects, AO , M, N, D, Y, can be arranged in 5!
ways. Also A and O can be arranged in 2! ways. So total number of ways,
5!  2! = 120  2 = 240
Illustration
Find the number of ways in which 5 girls and 5 boys can be arranged a in row if boys and girls are
alternate.

2nd Floor, Opp. Domino’s, VIP Road, Zirakpur. Ph: 01762-351579, 9815301916, 9878170502
Chase your Dream Permutation and Combination Class : XI 5

Solution
First 5 girls can be arranged in 5! ways, i.e,
GGGGG or GGGGG
Now, if girls and boys are alternate, then boys can occupy places with “” mark in the diagram.
Hence, the total number of arrangements is
5!  5! + 5!  5! = 2  5!  5!

DO YOURSELF - 1
Subjective Questions
1. Number of ways in which letters of the word COUNTRY be arranged ?
2. Number of ways in which letters of the word PENCIL be arranged such that word start with a vowel ?
3. Number of words which can be formed by using all the letters of the word CONFIRM such that F is in
the middle portion ?
4. How many ways may n girls and n boys be seated in a row of 2n chairs, if the two genders must
alternate ?
5. Find the total number of ways of selections of 4 distinct red balls, 2 identical black balls, and 3 identical
white balls.
Objective Questions
1. There are 5 roads leading to a town from a village. The number of different ways in which a villager
can go to the town and return back, is
(A) 25 (B) 20 (C) 10 (D) 5
2. How many words can be formed from the letters of the word BHOPAL
(A) 124 (B) 240 (C) 360 (D) 720
3. How many numbers can be formed from the digits 1, 2, 3, 4 when the repetition is not allowed
(A) 4 P4 (B) 4 P3 (C) 4 P1  4 P2  4 P3 (D) 4 P1 4 P2 4 P3 4 P4
4. How many numbers lying between 500 and 600 can be formed with the help of the digits 1, 2, 3, 4, 5,
6 when the digits are not to be repeated
(A) 20 (B) 40 (C) 60 (D) 80
5. 4 buses runs between Bhopal and Gwalior. If a man goes from Gwalior to Bhopal by a bus and comes
back to Gwalior by another bus, then the total possible ways are
(A) 12 (B) 16 (C) 4 (D) 8
6. In how many ways can 10 true-false questions be replied
(A) 20 (B) 100 (C) 512 (D) 1024
7. There are 8 gates in a hall. In how many ways a person can enter in the hall and come out from a
different gate
(A) 7 (B) 8 × 8 (C) 8 + 7 (D) 8 × 7
8. P, Q, R and S have to give lectures to an audience. The organiser can arrange the order of their
presentation in
(A) 4 ways (B) 12 ways (C) 256 ways (D) 24 ways
9. The product of any r consecutive natural numbers is always divisible by IIT 1985]
2 n
(A) r ! (B) r (C) r (D) None of these
10. The number of ways in which first, second and third prizes can be given to 5 competitors is
(A) 10 (B) 60 (C) 15 (D) 125
11. In a railway compartment there are 6 seats. The number of ways in which 6 passengers can occupy
these 6 seats is
(A) 36 (B) 30 (C) 720 (D) 120
12. If any number of flags are used, how many signals can be given with the help of 6 flags of different
colours
(A) 1956 (B) 1958 (C) 720 (D) None of these
13. The number of words which can be formed from the letters of the word MAXIMUM, if two consonants
cannot occur together, is
(A) 4 ! (B) 3 ! × 4 ! (C) 7 ! (D) None of these
14. The number of ways in which the letters of the word TRIANGLE can be arranged such that two vowels
do not occur together is
(A) 1200 (B) 2400 (C) 14400 (D) None of these

2nd Floor, Opp. Domino’s, VIP Road, Zirakpur. Ph: 01762-351579, 9815301916, 9878170502
Chase your Dream Permutation and Combination Class : XI 6

15. How many words can be formed form the letters of the word COURTESY, whose first letter is C and
the last letter is Y
(A) 6 ! (B) 8 ! (C) 2 (6) ! (D) 2 (7) !
16. How many words can be made from the letters of the word DELHI, if L comes in the middle in every
word
(A) 12 (B) 24 (C) 60 (D) 6
17. How many words can be made from the letters of the word BHARAT in which B and H never come
together
(A) 360 (B) 300 (C) 240 (D) 120
18. How many words can be made from the letters of the word INSURANCE, if all vowels come together
(A) 18270 (B) 17280 (C) 12780 (D) None of these
(B)
19. In how many ways can 5 boys and 5 girls stand in a row so that no two girls may be together
(A) (5!)2 (B) 5! 4! (C) 5!  6! (D) 6  5!
20. The number of arrangements of the letters of the word BANANA in which two N's do not appear
adjacently is
(A) 40 (B) 60 (C) 80 (D) 100
21. The number of words that can be formed out of the letters of the word ARTICLE so that the vowels
occupy even places is
(A) 36 (B) 574 (C) 144 (D) 754
22. The number of ways in which three students of a class may be assigned a grade of A, B, C or D so
that no two students receive the same grade, is
4 3
(A) 3 (B) 4 (C) 4 P3 (D) 4C 3
23. Using all digits 2, 3, 4, 5, 6 how many even numbers can be formed
(A) 24 (B) 48 (C) 72 (D) 120
24. In how many ways can 10 soldiers stand in two rows having 5 soldiers in each row ?
(A) 3628800 (B) 7257600 (C) 35400 (D) 985000
25. Number of ways in which the letters of the word RAINBOW be arranged such that N and B are
together is
(A) 2560 (B) 1540 (C) 540 (D) 1440
26. Number of ways in which 15 different books can be arranged on a shelf so that two particular books
shall not be together is
(A) 14  15! (B) 13  14! (C) 14!  15!
2
(D) (15!)
27. The number of arrangements of the letter of the word PAPAYA in which the two ‘P’ do not appear
adjacently is
(A) 40 (B) 60 (C) 80 (D) 36
28. How many different nine digit numbers can be formed with the number 223355888 by rearranging its
digits so that the odd digits occupy even positions ?
(A) 16 (B) 36 (C) 60 (D) 180
29. Number of outcomes, when 3 identical dice are thrown together, is
(A) 216 (B) 156 (C) 56 (D) 20
Answer key
Subjective Answers
2
1. 7! 2. 240 3. 720 4. 2(n!) 5. 192
Objective Answers
1 2 3 4 5 6 7 8 9 10 11 12 13 14 15
a d d a a d d d a b c a a c a
16 17 18 19 20 21 22 23 24 25 26 27 28 29
b c d c a c c c a d b a c c
6.2 Row Arrangement: Repeated objects
If you look at the word INDIA, there are 2 I’s in the word. Both I’s are identical, and it does not matter in
which order we write these two I’s, since they are same. In other word, if we exchange ‘I’ for ‘I’ you still
spell INDIA.
Now, if we arrange the letters ignoring the fact that two I’s are identical number of words formed will be 5!.

2nd Floor, Opp. Domino’s, VIP Road, Zirakpur. Ph: 01762-351579, 9815301916, 9878170502
Chase your Dream Permutation and Combination Class : XI 7

But in all these arrangements two I’s whenever they interchange their places keeping other letters fix, we
won’t get different word.
So relative position of I’s can be arranged in 2! ways and since two I’s are identical, hence total
5!
arrangement will be
2!
Similarly, if we have word RECEPTIVE containing three similar E’s, the total arrangements, say are X
and three E’s can be arrangement in 3! ways, so we have X  3! = 9!
9!
So X 
3!
So, number of arrangements of n different things taken all at a time. Where are alike of one kind, q are
n!
alike of second kind r are alike of third kind and the rest are different is given by .
p !q ! r !
11!
# Again, number of ways of arranging letters of the word ENGINEERING
3!3!2!2!
8!
# Similarly, ways of arranging letters of word BAHUBALI is
2!2!
4!
# Now if vowels are at even places, number of ways of arranging vowels are
2!
4!
And consonants are odd places
2!
4! 4!
So total arrangements: 
2! 2!
Here we come up with another conceptual example.
We have 7 digits 0,0,2,5,6,6,6.
Now if we wish to create seven digit numbers out of these given digits.
7!
What is the answer ?  A Big NO, it is incorrect.
2!3!
First place cannot be occupied by 0.
So first place can be filled either by 2,5 or 6.
5!
But wait, if first plane is occupied by 2 or 5 remaining places will be filled in
2!3!
5!
So required no of ways 2 
2!3!
5!
Again if first place is fitted by 6, remaining plane can be filled in
2!2!
5! 5!
So total number of ways 2  
2!3! 2!2!
Here the problem is dealt in two cases, so the addition rule is applicable to get the final answer

As shown in fig. a mouse at a is staring at cheese at B. In order to reach quickly, he examines the
shortest path to reach B. Length of each segment of grid is equal.

B
In how many ways he can reach at B.
After scanning the problem closely, we can clearly see mouse has to travel 4 units south and 3 units
towards east. For example, if he follows the dotted path, his pattern will be WSSWSSE. (W stands for
one unit towards west and S stands for one unit towards south).

2nd Floor, Opp. Domino’s, VIP Road, Zirakpur. Ph: 01762-351579, 9815301916, 9878170502
Chase your Dream Permutation and Combination Class : XI 8

7!
Now the number of ways will be equal to the all the permutation of 3 Ws and 4S’s . which can be
3!4!
ways.
W W W
A
S
S
S
S
B
Number of Permutations with Repetition.
(1) The number of permutations (arrangements) of n different objects, taken r at a time, when each object
may occur once, twice, thrice,........upto r times in any arrangement = The number of ways of filling r
places where each place can be filled by any one of n objects.
r – places :
Number of choices : 1 2 3 4 r
n n n n n
The number of permutations = The number of ways of filling r places = (n)r
(2) The number of arrangements that can be formed using n objects out of which p are identical (and of one
kind) q are identical (and of another kind), r are identical (and of another kind) and the rest are distinct is
n!
.
p!q!r!
Illustration
The number of arrangement of the letters of the word “CALCUTTA”
(A) 2520 (B) 5040 (C) 10080 (D) 40320
Solution: (B)
8!
Required number of ways   5040 . [since here 2C’s, 2T’s and 2A’s]
2 ! 2! 2!
Illustration
How many words can be made from the letters of the word ‘COMMITTEE’
9! 9! 9!
(A) (B) (C) (D) 9!
(2 !)2 (2 !)3 2!
Solution: (B)
9! 9!
Number of words =  [Since here total number of letters is 9 and 2M’s, 2T’s and 2E’s]
2!2!2! (2!)3
Concept Execution
(A) Rank in a Dictionary :
If the letters of the word FOOTBALL are arranged randomly. What is the rank of the word FOOTFALL.
If all the five digit numbers are formed by using the digits 2,2,0,1,5, are written in increasing orders, what is
the rank of 502012.
Numbers whose first digit is one :
4!
Fixing 1 at first place 1 _ _ _ _ ,we have i.e, 12 numbers.
2!
Again, fixing 2 at first place 2 _ _ _ _ , we have 4! i.e, 24 nos.
Now fixing 5 at first place. 5 _ _ _ _
3!
Numbers where first two digits are 50 _ _ _ are
2!
3!
Numbers where first two digits are 51 _ _ _ are
2!
Now fixing 2 at second place 5 2 _ _ _
Again 0 at third place 5 2 0 _ _
Next number will be obviously 52012
So the rank is 12 + 24 + 3+ 3 + 1 = 43.

2nd Floor, Opp. Domino’s, VIP Road, Zirakpur. Ph: 01762-351579, 9815301916, 9878170502
Chase your Dream Permutation and Combination Class : XI 9

(B) Sum of Numbers:


If we want to add all the five digit numbers which can be formed by using 2,2,3,6,8.
Let us start with the basics of addition. When we add the numbers we first start with adding all the digits at
unit place then at tenth place and so on. We will apply the same method but in an innovative way.
Now fixing 2 at unit place, other digits can be permutated in 4! ways. So value of all 2’s at unit place is 2  4!.
4! 4!
Again fixing 3 at unit place other digits can be arranged in ways, so value of all 3’s at unit place. Is 3 
2! 2!
So value of all digits at unit place is
4!
2  5!  (3  6  8) …. (i)
2!
Similarly value of all digits when they appear at tenth place is,
 5! 
2  5! 2!  (3  6  8)  10 … (ii)
 
Similarly at hundredth place
 5! 
2  5! 2!  (3  6  8)  100 … (iii)
 
At thousand place
 5! 
2  5! 2!  (3  6  8)  1000 …. (iv)
 
An finally at then thousand place
 5! 
2  5! 2!  (3  6  8)  10000 …. (v)
 
Now find sum by adding i, ii, iii, iv & v, we have
5!
(1+10+100+1000+10000) (2  5! + (3 + 6 +8)]
2!
(C) De-Arrangements:
Consider n distinct objects, a1, a2 …., an, arranged in a row in the following order
a1 a2 a3 a4 a5 …… an .
A de-arrangement of these objects is a permutation in which no object is in its original position
i.e, a1 is not in first place, a2 is not in the second plane, ……, an in not in nth place.
This concept is explained by inclusion – exclusion principle.
Consider a Venn diagram, shown in fig. total element in the universal set is N and in set X is b. Then
a=N–b

x
b
a
Again if three are two set, X and Y,
x y

b c d
a
a = N – (b + c + d)
or a = N–(b + c +d + c – c)
or n(X’  Y’) = N – (n(X) + n(Y) – n(X  Y)) ( Number of elements neither in X nor Y )
Also, for three set,
N ( X  Y  Z) = N– (n(X) + n(Y) + n(Z) – (n(X  Y) – n(Y  Z) ) – n(X  Z))
This is principle of inclusion – exclusion
Let us understand in a lucid (easy) way with an example

 On a rainy day 3 people go to a party in a hotel. Each of them leaves his umbrella at the counter. Now
number of ways in which the umbrellas are handed back to them after the party in such a way that no
person receives his own umbrella.
As shown in Venn diagram, we want to find out

2nd Floor, Opp. Domino’s, VIP Road, Zirakpur. Ph: 01762-351579, 9815301916, 9878170502
Chase your Dream Permutation and Combination Class : XI 10

n(ABC) i.e, n(U) – n(A  B C)


Or Total ways – (at least one receives his hat)
i.e, N – (n(A) + n(B) + n(C) – (n(A  B) + n(B  C) + n(C  A)) + n(A  B  C) … (i)
Now total ways of distributing hat to each one
i.e, N = 3! .… (ii)
Number of ways of distributing hat such that one receives A B
their hat i.e, n(A) + n(B) + n(C) = C1  2!
3

3! 3!
or  2!  …. (iii)
2!1! 1! A&B B receive
A receive Receive his hat
Again, number of ways of distributing such that two of them his hat
receive their hat
i.e, n(A  B) + n(B  C) + n(C  A) = C2  1!
3 No one A&C B&C
3! 3! receive Receive Receive
=  1!  …. (iv) their hat
1!2! 2!
Lastly number of ways of distributing such that all of them C receive
receives their hat his hat
3! 3!
i.e, n(A  B  C) = C3
3
=  .... (v) C
3!0! 3!
using (i) from (ii), (iii), (iv), (v) we have
 3! 3! 3!   1 1 1
n(A  B  C) = 3! –     = 3!  1    
 1! 2! 3!   1! 2! 3! 
 This is a big result and is applicable for any number of objects.
Let us say, we have 4 balls of different colours, i.e, red, green, blue, green and four boxes of same
colours. If we went to distribution each ball in each box such that no ball goes to box of same colour, then
the number of ways will be,
 1 1 1 1
= 4!  1     
 1! 2! 3! 4! 
So, if we have to de-arrange ‘n’ objects say a1, a2 , ….. an which are arranged in the following order
a1 a2 a3 …. an.
A de-arrangement of these objects such that no one of these objects goes to its original place, then the
number of ways will be
 1 1 1 1
n !  1     ......  (1)n 
 1! 2! 3! n! 
Illustration
Find the number of permutation of all the letters of the word MATHEMATICS which starts with
consonants only.
Solution
(M M), (A A), (T T), H, E, I, C, S
10!
Words starting with M or A or T are
2!2!2!
10! 10! 10!  5  11!
Hence, number of words are 3 5   3  
2!2! 2!2!2! 2!2!  2 8
Illustration
There are four balls of different colours and four boxes of colurs same as those of the balls. The number
of ways in which the balls, one in each box, could be placed such that a ball doesn't go to box of its own
colour is
(A) 8 (B) 7 (C) 9 (D) None of these
Solution: (C)
1 1 1
Number of derangement are = 4 !      12  4  1  9 .
 2 ! 3 ! 4 !
 1 1 1 1 1
(Since number of derangements in such a problem is given by n ! 1     ......  (1)n  .
 1! 2! 3! 4 ! n!

2nd Floor, Opp. Domino’s, VIP Road, Zirakpur. Ph: 01762-351579, 9815301916, 9878170502
Chase your Dream Permutation and Combination Class : XI 11

Illustration
The number of ways in which the letters of the word ARRANGE be arranged so that
(i) the two R’s are never together,
(ii) the two A’s are together but not two R’s
(iii) neither two A’s nor two R’s together.
Solution
The letters of word ARRANGE can be rewritten as ARNGE
AR
So we have 2A’s and 2R’s i.e, total 7 letters.
(i) Total number of words is 1/x + 1/y = 1/n.
The number of words in which 2 R’s are together [consider (R R) as one unit] is 6!/2! E.g.,
(RR) A, A, N, G, E
Note that permutations of RR give nothing extra. Therefore, the number of words in which the two
7! 6!
R’s are never together is   900
2!2! 2!
(ii) The number of words in which both A’s are together is 6!/2! = 360, e.g.,
(AA), R, R, N, G, E
The number of words in which both A’s and both R’s are together is 5! = 120, e.g.,
(A A), (R R), N, G, E
Therefore, the number of words in which both A’s are together but the two R’s are not together is
360–120 = 240.
(iii) There are in all 900 words in each of which the two R’s are never together. Consider any such word.
Either the two A’s are together or the two A’s are not together. But the number of all such
arrangements in which the two A’s are together is 240. Hence, the number of all such arrangements
in which the two A’s are not together is 900–240 = 660.

DO YOURSELF -2
Subjective Questions
1. Find the number of words formed by permuting all letters of the word SPARDHA. In how many of them
vowels are together?
2. How many words can be formed using the letters E twice, the letter C thrice and the letter P twice ?
3. Find the number of words formed by permuting all letters of the word INDIA. In how many of them
vowels are together?
4. Find the number of different words formed with the letters of the word HARYANA.
5. In how many ways can the letters of the word INSURANCE be arranged, so that vowels are occupied
at even places?
6. Find the rank of the word ‘Kanpur’ when the letters of word ‘Kanpur’ is arranged in all possible ways
and written in dictionary.
7. If all the meaningful or meaningless words formed from all the letters of the word ‘OLYMPIAD’ taken
one at a time be arranged in alphabetical dictionary ordering, then
(i) how many words are possible ?
(ii) What is the first word?
(iii) What is the last word ?
(iv) how many words begin with O?
(v) What is the serial number of the word ‘OLYMPIAD’ in the dictionary ordering ?
Objective Questions
1. The number of permutations of the letters x 2 y 4 z 3 will be
9! 9! 9!
(A) (B) (C) (D) 9 !
2!4! 2!4!3! 4!3!
2. How many numbers consisting of 5 digits can be formed in which the digits 3, 4 and 7 are used only
once and the digit 5 is used twice
(A) 30 (B) 60 (C) 45 (D) 90
3. The number of different arrangements which can be made from the letters of the word SERIES taken
all together is
6! 6!
(A) (B) (C) 6 ! (D) None of these
2!2! 4!

2nd Floor, Opp. Domino’s, VIP Road, Zirakpur. Ph: 01762-351579, 9815301916, 9878170502
Chase your Dream Permutation and Combination Class : XI 12

4. How many words can be formed with the letters of the word MATHEMATICS by rearranging them
11! 11! 11!
(A) (B) (C) (D) 11 !
2!2! 2! 2!2!2!
5. How many words can be made out from the letters of the word INDEPENDENCE, in which vowels
always come together
(A) 16800 (B) 16630 (C) 1663200 (D) None of these
6. In how many ways 5 red, 4 blue and 1 green balls can be arranged in a row
(A) 1260 (B) 2880 (C) 9 ! (D) 10 !
7. The number of ways of arranging the letter AAAAA BBB CCC D EE F in a row when no two C’s are
together is
13
15! 15! 13! 12! P3 12!
(A)  3! (B)  (C)  (D)  13P3
5!3!3!2! 5!3!3!2! 5!3!2! 5!3!2! 3! 5!3! 2!
8. Using 5 conveyances, the number of ways of making 3 journeys is
(D) 5  1
5 3 3
(A) 3 × 5 (B) 3 (C) 5
9. The total number of permutations of the letters of the word “BANANA” is
(A) 60 (B) 120 (C) 720 (D) 24
10. The number of 7 digit numbers which can be formed using the digits 1, 2, 3, 2, 3, 3, 4 is
(A) 420 (B) 840 (C) 2520 (D) 5040
11. The number of 3 digit odd numbers, that can be formed by using the digits 1, 2, 3, 4, 5, 6 when the
repetition is allowed, is
(A) 60 (B) 108 (C) 36 (D) 30
12. How many different nine-digit numbers can be formed from the digits of the number 223355888 by
rearrangement of the digits so that the odd digits occupy even places
(A) 16 (B) 36 (C) 60 (D) 180
13. The number of ways in which 6 rings can be worn on the four fingers of one hand is
6 4
(A) 4 (B) 6 C 4 (C) 6 (D) None of these
14. In how many ways can 4 prizes be distributed among 3 students, if each student can get all the 4 prizes
(C) 3  1
4 4 3
(A) 4 ! (B) 3 (D) 3
15. In how many ways 3 letters can be posted in 4 letter-boxes, if all the letters are not posted in the same
letter-box
(A) 63 (B) 60 (C) 77 (D) 81
16. There are 4 parcels and 5 post-offices. In how many different ways the registration of parcel can be
made
(D) 5  4
5 4 4 5
(A) 20 (B) 4 (C) 5
17. The number of ways in which the letters of the word ARRANGE can be arranged such that both R do
not come together is
(A) 360 (B) 900 (C) 1260 (D) 1620
18. The number of ways in which all four S come consecutively in the word MISISSIPPI, is
(A) 420 (B) 840 (C) 210 (D) 630
19. The number of words that can be formed from the letters of the word ‘SEREIES’ which start with S and
end with S are
(A) 24 (B) 48 (C) 12 (D) 16
20. If all the letters of the word ‘AGAIN’ be arranged as in a dictionary, then the fiftieth word is
(A) NAAGI (B) NAAIG (C) NIAAG (D) NAIAG
21. The number of ways in which 5 letters can be placed in 5 marked envelopes, so that no letter is in the
right envelope
(A) 45 (B) 44 (C) 43 (D) 46
22. In a class of 10 students, roll numbers are written on their respective chairs. Find the ways in which
exactly 4 students do not sit on their chairs.
(A) 1890 (B) 1980 (C) 1650 (D) 1350
23. Four children are playing hide and seek. Find the total number of ways in which no one gets into their
respective house
(A) 9 (B) 8 (C) 7 (D) 6
24. Let A = {x1, x2, x3} and B = {y1 , y2, y3} then the number of one-one mapping from A to B such that
F(xi)  yi, i = 1,2,3
(A) 2 (B) 3 (C) 4 (D) 5

2nd Floor, Opp. Domino’s, VIP Road, Zirakpur. Ph: 01762-351579, 9815301916, 9878170502
Chase your Dream Permutation and Combination Class : XI 13

25. How many different words can be made with the letters of the word NAINITAL such that each of the
word begins with L and end with T?
(A) 85 (B) 88 (C) 92 (D) 90
26. There are 6 books of physics, 3 of chemistry and 4 of biology. Number of ways in which these books
be placed on a shelf if the books of the same subject are to be together is
(A) 622080 (B) 888000 (C) 222000 (D) 413080
27. How many 6 digit numbers can be formed out of the digits of the number 113226?
(A) 156 (B) 180 (C) 280 (D) 120
28. Number of ways in which the letters of the word TAMANNA be arranged is
(A) 120 (B) 420 (C) 840 (D) 500
29. Number of 6 digit numbers that can be formed using the digit 2 two times and the digit 5 four times is
(A) 16 (B) 15 (C) 24 (D) 18
30. Number of way in which 15 billiard balls be arranged in a row if 3 are identical red, 7 are identical white
and remaining are identical black balls is
15! 15!
(A) 15! (B) (C) 7!3!5! (D)
7!3!5! 7!5!
31. Number of ways in which 5 plus (+) signs and 5 minus (–) signs be arranged in a row so that no two
minus signs are together is
(A) 6 (B) 7 (C) 8 (D) 10
Answer key
Subjective Answers
1. 2520, 720 2. 210 3. 60, 18 4. 840
5. 1440 6. 122 7. (i) 40320; (ii) ADILMOPY; (iii) YPOMLIDA; (iv) 5040; (v) 28055

Objective Answers

1 2 3 4 5 6 7 8 9 10 11 12 13 14 15
b b a c a a c c a a b c a b B
16 17 18 19 20 21 22 23 24 25 26 27 28 29 30
c b b C b b a a a d a b b b b
31
a

6.3 Combination
We have discussed earlier so far, the arrangements of objects in a row, distinct as well as identical.
Now here we have landed on a new word COMBINATION. What exactly is combination? Combination
means forming a group or team taken some or all of them at a time.
Let’s say we have first five alphabets a,b,c,d,e and we want to select three of them.
Now for selecting the first one we have 5 options and for second one we have 4 options and for third are
we have 3 options.
So, 5  4  3 = 60 is the answer?
Let us write all the possible selections, a,d,e, a,c,e, acd , abc, abd, abe, bdc, bce, bde, cde. As we can
see there are only 10 combinations. So where we actually went wrong? Record it, Note it, Memorize it
combination or forming a group does not include arrangement i.e, abc or acb or bca are treated only one
selection.
So when we multiplied 5  4 3 ways of selecting, unknowingly we arranged the things as well. So if we
divide this selection 5  4  3 which includes arrangement as well by 3! As we treated all six arrangement
i.e, abc, acb, bca, bac, cab, cba as one selecting we will get the correct answer i.e,
543
 10
3!
Now if we have ‘n’ distinct objects and we want to select ‘r’ objects so approaching in the same manner,
rd th
i.e, first selection in n ways, second in (n–1) ways, 3 in (n–2) ways, and r in n–r ways and diving by r!
we have,

2nd Floor, Opp. Domino’s, VIP Road, Zirakpur. Ph: 01762-351579, 9815301916, 9878170502
Chase your Dream Permutation and Combination Class : XI 14

n.(n  1)(n  2).....(n  r ) n (n  1)(n  2)...(n  r )(n  r  1).....2.1


Or
r! r !(n  r  1)(n  r  2).....2.1
n! n
Or which is denoted as Cr .
r !(n  r )!
Hey; we have very concise and uncomplicated result.
If we need to select or form a group of r objects out of n distinct objects then the number of ways are
n!
n
Cr 
n  r !r !
5 5!
So, selecting 3 out of 5 distinct alphabet is   10
3 3!2!
10!
 Again selecting 4 volunteers from 10 available boys , is 10 C 4   210
4!6!
Note, each combination is considered different even if we fix 3 and replace only one out of 4. So, abcd
and abce are different.
 Again if three 15 probable for Indian cricket team and we want to select 11 players there number of ways
15
will be C11 now, if the condition is out of 15 probable, Captain Virat is to be always included, total ways
14
will be C10
Excluding Virat, as he is already included, we had selected 10 out of remaining 14.
 Also if , one player is out of form, say RISHAV PANT and is to excluded from the team then the number
14
of ways of selecting 11 Out of 15 probable is C11.
Here, excluding RISHAV, we have selected 11 out of 14.
 Let us navigate to more examples. If from the letters of the word EDUCATION. We have to select 5
letters such that exactly three are vowels and two are consonants.
Now we have 5 vowels & 4 consonants in the word EDUCATION.
5
So selecting vowels in C3 ways.
4
AND selecting consonants in C2 ways.
Now since we have to select both simultaneously so the concept of AND is used, total selections are in
C3  C2 ways.
5 4

 Out of 8 girls if we want to select 5, such that two particular girls SWEETY and NAUGHTY does not wish
to come together.
Now we can approach this problem in two ways
st
1 Method:
No. of ways
= selection including SWEETY excluding NAUGHTY + selection including NAUGHTY excluding
SWEETY + Selection excluding both
6 6 6
= C4 + C4 + C5
nd
2 Method
No. of ways = Total selection – Selection including both
8 6
= C5 – C3
 How many ways are there of choosing 6 cards containing an ace and a king from a pack of 52 playing
cards?
Now this is conditional selection.
An ace AND a king AND 4 other Cards

A K

= C1  C1  C4
4 4 44

 A committee of 4 is to be formed out of 6 boys and 4 girls such that it includes at least one girl.
st
1 Method
Possible Cases are:
1 Girl 3 Boys 2 Girls 2 Boys

  
 
3 Girls
or
1 Boy 4 Girls


or    or

2nd Floor, Opp. Domino’s, VIP Road, Zirakpur. Ph: 01762-351579, 9815301916, 9878170502
Chase your Dream Permutation and Combination Class : XI 15

= C1  C3 + C2  C2 + C3  C1 + C4 .
4 6 4 6 4 6 4

IInd Method
No. of ways = Total selections – No girl is selected
10 6
= C4 – C4

Illustration
In the 13 cricket players 4 are bowlers, then how many ways can form a cricket team of 11 players in
which at least 2 bowlers included
(A) 55 (B) 72 (C) 78 (D) None of these
Solution: (C)
The number of ways can be given as follows:
2 bowlers and 9 other players = 4 C2 9 C9 ; 3 bowlers and 8 other players = 4 C3 9 C8
4 bowlers and 7 other players = 4 C4 9 C7
Hence required number of ways = 6 × 1 + 4 × 9 + 1 × 36 = 78.
Illustration
In how many ways a team of 10 players out of 22 players can be made if 6 particular players are always
to be included and 4 particular players are always excluded
(A) 22 C10 (B) 18 C3 (C) 12 C4 (D) 18 C4
Solution: (C)
6 particular players are always to be included and 4 are always excluded, so total number of selection,
now 4 players out of 12.
Hence number of ways = 12 C4 .
Illustration
In how many ways can 6 persons to be selected from 4 officers and 8 constables, if at least one officer is
to be included
(A) 224 (B) 672 (C) 896 (D) None of these
Solution: (C)
Required number of ways = 4 C1  8 C5 4 C2  8 C4 4 C3 8 C3  4 C4  8 C2 = 4 × 56 + 6 × 70 + 4 × 56 + 1 × 28 =
896.
Concept Execution
Geometrical Application :
(i) Number of Lines: If there are n non-collinear points in a plane, the total number of straight lines
n 6
obtained by joining these n points is C2. As shown in fig. 6 non-collinear points can be joined by C2 = 15
lines.
Each line is formed by selecting any two points.

 If m points out of n points are collinear, than the total number of straight lines obtained by joining these
points will be

m-collinear
points

n m
C2 – C 2 + 1
m
Note C2 lines which are excluded as we can get only one line by joining m-collinear

2nd Floor, Opp. Domino’s, VIP Road, Zirakpur. Ph: 01762-351579, 9815301916, 9878170502
Chase your Dream Permutation and Combination Class : XI 16

 As shown in fig. a tetrarchy’s (a triangle with 10 dots) is given number of fresh lines which can be formed
by joining any two dots.

There are 3 sets of 4 collinear points and 3 sets of 3 collinear points. So, number of line which can be
formed are
= C2 – 3  C2 + 3 – 3  C2 + 3
10 4 3

3 collinear
points

4 collinear
points
(ii) Number of Triangles: If out of ‘n’ non-collinear points in a plane. If, we want to find total number of
triangles by joining these points then we have to select 3 distinct points. Each selection will give a
different triangle
n
So, number of triangles will be C3.
n m
Again if ‘m’ points are collinear then number of triangles formed will be C3 – C3.

(iii) Number of Diagonals:


As shown in fig. number of diagonals found by joining vertices of octagon can be found by selecting any
two vertices. But selection will also include number of sides as well so total diagonals are

8
= C2 – 8
 Number of diagonals in polygon of n-sides.
n n (n  3)
= C2 – n =
2
(iv) Number of Parallelograms:
If m parallel lines in a plane are intersected by a family of lines of other n parallel lines the total number of
parallelogram so formed.
= C2  C2
m n

2nd Floor, Opp. Domino’s, VIP Road, Zirakpur. Ph: 01762-351579, 9815301916, 9878170502
Chase your Dream Permutation and Combination Class : XI 17

Concept: To form a parallelogram we need two parallel lines from one family and two parallel lines from
another family. So selection each selection of four lines C2  C2 will generate one parallelogram.
m n

 If the families are mutually perpendicular then C2  C2 rectangles will be formed.


m n

(v) Number of squares:


Consider a square divided into four unit squares no of squares formed will be
2  2  1 1
Smaller square Bigger square

1 2

2
1
Similarly, if it is divided into nine unit squares, no of squares will be
33+22+11

3
So, in general 1  1 + 2  2 + 3  3 + ….. + n  n
Now, if we have a grid of unit squares.

3
Smallest squares will be 4  3. Squares of side two unit will be 32, square of side
three units will be 21
So total square are 4  3 + 3  2 + 2  1
So, in general if we have a grid of n  m
Then number of squares that can be formed
= n  m + (n – 1)(m – 1)+ (n –2) (m – 2) + ….
CRACK THIS. If there are 8 people standing on the circumference of circle, then number of ways in which
three people can be selected if no two are adjacent. A B
This problems can be dealt with the concept of number of triangle which can be
formed by joining vertices of a octagon such that no side of triangle is side of octagon. C
H
8
Total no. of selection = C3 … (i) D
If two people are located adjacent to each other i.e, A & B then the third person can G
be selected in 4 ways (Leaving H & C) F E
A B
So similarly if B & C are selected and so on, we have number of ways such that two
people are adjacent to each other C
8  4 = 32 ways … (ii) H
If all of them are consecutive than no of selection will be 8. …. (iii) D
So required number of ways using (i), (ii) & (iii) is G
8 E
C3 – 32 – 8 F
CRACK THIS Chords are formed by joining 10 points using on the circumference of the circles. If no three
chords are concurrent then number of intersecting points lying inside the circle will be ____.

2nd Floor, Opp. Domino’s, VIP Road, Zirakpur. Ph: 01762-351579, 9815301916, 9878170502
Chase your Dream Permutation and Combination Class : XI 18

Concept : If we select any four points out of given ten points say A,B, C and D and join them alternatively i.e.
A to C and B to D, we will get one intersecting point.

C
D

So, that simplifies the problem, selecting any four points out of ten given points will generate one intersecting
point using inside the circle.
10
Total number of intersecting points will be C4

FINER POINTS
(1) Number of total different straight lines formed by joining the n points on a plane of which m (< n) are
collinear is n C 2  m C 2  1 .
(2) Number of total triangles formed by joining the n points on a plane of which m (< n) are collinear is
n
C3 m C3 .
(3) Number of diagonals in a polygon of n sides is n C 2  n .
(4) If m parallel lines in a plane are intersected by a family of other n parallel lines. Then total number of
mn (m  1)(n  1)
parallelograms so formed is m
C 2  n C 2 i.e
4
(5) Given n points on the circumference of a circle, then
n n
(i) Number of straight lines = C2 (ii) Number of triangles = C3
n
(iii) Number of quadrilaterals = C 4 .
(6) If n straight lines are drawn in the plane such that no two lines are parallel and no three lines are
concurrent. Then the number of part into which these lines divide the plane is = 1  n .
n
(7) Number of rectangles of any size in a square of n  n is r
r 1
3
and number of squares of any size

n
is r
r 1
2
.

np
(8) In a rectangle of n  p (n  p) number of rectangles of any size is (n  1)( p  1) and number of
4
n
squares of any size is  (n  1  r)(p  1  r) .
r 1

Illustration
The number of diagonals in a octagon will be
(A) 28 (B) 20 (C) 10 (D) 16
Solution: (B) Number of diagonals = C 2  8 = 28  8 = 20.
8

Illustration
The number of straight lines joining 8 points on a circle is
(A) 8 (B) 16 (C) 24 (D) 28
8
Solution: (D) Number of straight line = C 2 = 28.
Illustration
The number of triangles that can be formed by choosing the vertices from a set of 12 points, seven of
which lie on the same straight line, is
(A) 185 (B) 175 (C) 115 (D) 105
Solution: (A)
Required number of ways = 12 C 3  7 C 3 = 220 – 35 = 185.
Illustration

2nd Floor, Opp. Domino’s, VIP Road, Zirakpur. Ph: 01762-351579, 9815301916, 9878170502
Chase your Dream Permutation and Combination Class : XI 19

Out of 18 points in a plane, no three are in the same straight line except five points which are collinear.
The number of (i) straight lines (ii) triangles which can be formed by joining them
(i) (A) 140 (B) 142 (C) 144 (D) 146
(ii) (A) 816 (B) 806 (C) 800 (D) 750
Solution: (c, b)
Out of 18 points, 5 are collinear
(i) Number of straight lines = 18 C2 5 C2  1  153  10  1  144
(ii) Number of triangles  18C3 5 C3  816  10  806 .
Illustration
Find the maximum number of points of intersection of 6 circles.
Solution
6
Two circles intersect maximum at two distinct points. Now, two circles can be selected in C2 ways.
Again, each selection of two circles gives two points of intersection. Therefore, the total number of points
of intersection is C2  2 = 30.
6

Illustration
Find the total number of rectangles on the normal chessboard.
Solution
To form a rectangle on a chessboard two vertical lines and two horizontal lines should be selected. There
are 9 vertical lines and 9 horizontal lines found on the chessboard. Selection of 2 vertical and 2 horizontal
lines can be done in C2  C2 ways, which is equivalent to the number of rectangles.
9 9

DO YOURSELF -3
Subjective Questions
1. If there are 12 persons in a party, and if each two of them shake hands with each other, how many
hand shakes happen in the party?
2. Out of 15 balls, of which some are white and the rest are black, how many should be white so that the
number of ways in which the balls can be arranged in a row may be the greatest possible? It is
assumed that the balls of same colour are alike.
3. Find the number of ways in which the birthdays of six different persons will fall in exactly two calendar
months.
4. Four visitors A, B, C, D arrived at a town that has 5 hotels. In how many ways, can they disperse
themselves among 5 hotels.
5. Out of 8 sailors on a boat, 3 can work only on one particular side and 2 only on the other side. Find the
number of ways in which the sailors can be arranged on the boat.
6. Find the number of all three elements subsets of the set {a1, a2, a3…. an} which contain a3.
7. An automobile manufacturer has four colours available for automobile exteriors and three for interiors.
How many different colour combinations can be produce ?
8. How many number of ways of choosing 4 card from a pack of 52 playing card?
9. How many selections of 4 balls can be made from 5 red balls and 4 green balls if balls of the same
colour are different in size ?
10. At spardha, how many ways can a student choose a programme of 5 courses if 9 courses are available
and 2 specific courses are compulsory for even student ?
11. Find the number of square of any size from a square of size 4  4.
12. The number of rectangles of any size in a square of 5  5 is ____
13. A polygon has 8 sides. Then find the number of triangles having one side common with the sides of
polygon.
14. If PQRS is a convex quadrilateral with 3,4,5 and 6 points marked on sides PQ, QR, RS and PS
respectively, then find the number of triangle with vertices on different side.

Objective Questions
1. There are 12 volleyball players in all in a college, out of which a team of 9 players is to be formed. If
the captain always remains the same, then in how many ways can the team be formed
(A) 36 (B) 108 (C) 99 (D) 165
2. There are 16 vacancies for clerks in a certain office, 20 applications are received. In how many ways
can the clerks be appointed
(A) 3800 (B) 3876 (C) 969 (D) 4845

2nd Floor, Opp. Domino’s, VIP Road, Zirakpur. Ph: 01762-351579, 9815301916, 9878170502
Chase your Dream Permutation and Combination Class : XI 20

3. In how many ways a committee of 5 members can be formed out of 8 gentlemen and 4 ladies, if one
particular lady is always to be taken
(A) 140 (B) 330 (C) 560 (D) None of these
4. How many words can be formed by taking 3 consonants and 2 vowels out of 5 consonants and 4
vowels
5
C 3  4C 2
(A) 5 C3  4C2 (B) (C) 5 C3 4C3 (D) (5 C3  4C2 )(5)!
5
5. A male and a female typist are needed in an institution. If 10 ladies and 15 gentlemen apply, then in
how many ways can the selection be made
(A) 125 (B) 145 (C) 150 (D) None of these
6. Everybody in a room shakes hand with everybody else. The total number of hand shakes is 66. The
total number of persons in the room is
(A) 11 (B) 12 (C) 13 (D) 14
7. There are 9 chairs in a room on which 6 persons are to be seated, out of which one is guest with one
specific chair. In how many ways they can sit
(A) 6720 (B) 60480 (C) 30 (D) 346
8. On the occasion of Deepawali festival each student of a class sends greeting cards to the others. If
there are 20 students in the class, then the total number of greeting cards exchanged by the students is
(A) 20 C2 (B) 2.20 C2 (C) 2.20 P2 (D) None of these
9. In how many ways can 5 red and 4 white balls be drawn from a bag containing 10 red and 8 white
balls
(A) 8 C5 10C4 (B) 10 C5  8C4 (C) 18C9 (D) None of these
10. There are 15 persons in a party and each person shake hand with another, then total number of hand
shakes is
(A) 15 P2 (B) 15 C2 (C) 15! (D) 2(15!)
11. In a cricket championship there are 36 matches. The number of teams if each plays one match with
other are
(A) 8 (B) 9 (C) 10 (D) None of these

12. The number of triangles that can be formed by 5 points in a line and 3 points on a parallel line is
(A) 8 C 3 (B) 8 C3 5C3 (C) 8 C3 5C3  1 (D) None of these
13. The maximum number of points of intersection of 20 straight lines will be
(A) 190 (B) 220 (C) 200 (D) None of these
14. If a polygon has 44 diagonals, then the number of its sides are
(A) 7 (B) 11 (C) 8 (D) None of these
15. How many triangles can be drawn by means of 9 non-collinear points
(A) 84 (B) 72 (C) 144 (D) 126
16. The number of diagonals in a polygon of m sides is
1 1 1 1
(A) m (m  5) (B) m(m  1) (C) m(m  3) (D) m (m  2)
2! 2! 2! 2!
17. In a plane there are 10 points out of which 4 are collinear, then the number of triangles that can be
formed by joining these points are
(A) 60 (B) 116 (C) 120 (D) None of these
18. There are 16 points in a plane out of which 6 are collinear, then how many lines can be drawn by
joining these points
(A) 106 (B) 105 (C) 60 (D) 55
19. The number of parallelograms that can be formed from a set of four parallel lines intersecting another
set of three parallel lines is
(A) 6 (B) 18 (C) 12 (D) 9
20. The greatest possible number of points of intersection of 8 straight lines and 4 circles is
(A) 32 (B) 64 (C) 76 (D) 104
21. There are 16 points in a plane, no three of which are in a straight line except 8 which are all in a
straight line. The number of triangles that can be formed by joining them equals
(A) 504 (B) 552 (C) 560 (D) 1120

2nd Floor, Opp. Domino’s, VIP Road, Zirakpur. Ph: 01762-351579, 9815301916, 9878170502
Chase your Dream Permutation and Combination Class : XI 21

22. Let Tn denote the number of triangles which can be formed using the vertices of a regular polygon of
n sides. If Tn 1  Tn  21 then n equals
(A) 5 (B) 7 (C) 6 (D) 4
23. Out of 10 points in a plane 6 are in a straight line. The number of triangles formed by joining these
points are
(A) 100 (B) 150 (C) 120 (D) None of these
24. The number of straight lines that can be formed by joining 20 points no three of which are in the same
straight line except 4 of them which are in the same line
(A) 183 (B) 186 (C) 197 (D) 185
25. There are n distinct points on the circumference of a circle. The number of pentagons that can be
formed with these points as vertices is equal to the number of possible triangles. Then the value of n
is
(A) 7 (B) 8 (C) 15 (D) 30
26. Given six line segments of lengths 2, 3, 4, 5, 6, 7 units, the number of triangle that can be formed by
these lines is
(A) 6 C3  7 (B) 6 C3  6 (C) 6 C3  5 (D) 6 C3  4
27. A polygon has 35 diagonals, then the number of its sides is
(A) 8 (B) 9 (C) 10 (D) 11
28. If 5 parallel straight lines are intersected by 4 parallel straight lines, then the number of
parallelograms thus formed is
(A) 20 (B) 60 (C) 101 (D) 126
29. The maximum number of points of intersection of 8 circles, is
(A) 16 (B) 24 (C) 28 (D) 56
30. There are 10 points in a plane of which no three points are collinear and 4 points are concyclic. The
number of different circles that can be drawn through at least 3 points of these points is
(A) 116 (B) 120 (C) 117 (D) None of these
31. The number of ways lawn tennis mixed double can be made up from seven married couples if no husband
and wife play in the same set is
(A) 210 (B) 420 (C) 840 (D) None of these
32. A person has 5 shirts, 4 coats and 7 ties. Number of ways in which he can wear them is
(A) 100 (B) 140 (C) 96 (D) 124
33. In how many ways 7 member forming a committee out of 11 be selected so that 3 particular members
must be included?
(A) 81 (B) 70 (C) 95 (D) 125
34. In a meeting everyone had shaken hands with everyone else, it was found that 66 handshakes were
exchanged. Number of persons present in the meeting is
(A) 17 (B) 12 (C) 13 (D) 18
35. In a badminton tournament each player played one game with all the other players. Numbers of
players participated in the tournament if they played 105 games is
(A) 35 (B) 15 (C) 12 (D) 10
36. On the eve of new year each student of class XI sends cards to his classmates. If there are 40
students in the class number of cards exchanged during the process is
(A) 1200 (B) 600 (C) 780 (D) 1560
37. A polygon has 90 diagonals. Number of the sides is
(A) 25 (B) 17 (C) 15 (D) 14
38. There are m points on the line AB and n points on the line AC, excluding the point A. Triangles are
formed joining these points.
(i) When point A is not included. (ii) When point A is included
The ratio of the number of such triangles is
m n 2 m n 2 m n 2 m n 2
(A) (B) (C) (D)
2 2 m n m n
39. Ramya gives a dinner party for 5 guests. The number of ways in which they may be selected from
among 9 friends if two of the friends will not attend the party together is
(A) 84 (B) 133 (C) 91 (D) 126

2nd Floor, Opp. Domino’s, VIP Road, Zirakpur. Ph: 01762-351579, 9815301916, 9878170502
Chase your Dream Permutation and Combination Class : XI 22

40. The number of triangles that can be formed by 5 points in a line and 3 points on a parallel line is
8 8 5 8 5 5
(A) C3 (B) C3 – C3 (C) C3 – C3 – 1 (D) C3
41. The number of divisors of 10800 which are divisible by 15 is
(A) 10 (B) 20 (C) 15 (D) 30
42. How many triangles can be formed by using four points on a circle?
(A) 4 (B) 6 (C) 8 (D) 10
43. There are n points in a plane of which p points are collinear. How many lines can be formed from
these points?
n–p n p n p n p
(A) C2 (B) C2 – C2 (C) C2 – C2 + 1 (D) C2 – C2 – 1
44. The greatest possible number of points of intersection of 8 straight lines and 4 circles is
(A) 32 (B) 64 (C) 76 (D) 104
45. The number of parallelograms that can be formed from a set of four parallel lines intersecting another
set of three parallel lines is
(A) 6 (B) 18 (C) 12 (D) 9
46. A parallelogram is cut by two set of m lines parallel to its sides. The number of parallelogram thus
formed is
m 2 m+1 2 m+2 2 m+1
(A) ( C2) (B) ( C2) (C) ( C2) (D) C2
47. The number of triangles whose vertices are at the vertices of an octagon but none of whose sides
happen to come from the sides of the octagon is
(A) 24 (B) 52 (C) 48 (D) 16
48. A committee of 12 is to be formed from 9 women and 8 men. Then the number of ways of selecting
committees so that men are in majority is
(A) 3202 (B) 1134 (C) 2702 (D) 1314
49. A class has 15 students. The number of ways of forming a team of student including at least two
students and also excluding at least two students, is
(A) 32768 (B) 32736 (C) 16384 (D) 16368

Answer key
Subjective Answers
3. C6  6! 5. 3  (4!)
12 31 2
1. C2 2. 7008 4. 120
(n  2)(n  3)
6. 7. 12 8. 270725 9. 126 10. 35
2
11. 30 12. 225 13. 32 14. 342
Objective Answers

1 2 3 4 5 6 7 8 9 10 11 12 13 14 15
D D B D C B A B B B B C A B A
16 17 18 19 20 21 22 23 24 25 26 27 28 29 30
C B A B D A B A D B B C B D C
31 32 33 34 35 36 37 38 39 40 41 42 43 44 45
B B B B B D C C C C D A C D B
46 47 48 49
C D B B

6.4 Selection of one or more objects: (Total Selections)


n
We are aware now, Cr means selecting ‘r’ distinct objects out of ‘n’ distinct objects.
Now, we will be discussing about total selections i.e, all the possible ways of selection.
You have 8 friends. In how many ways you can host a birthday party by inviting one or more friends.
Now you have option for inviting one or more than one friends to party. So number of ways will be
8 8 8 8
C1 + C2 + ….. + C8 = 2 – 1
n n
This is total selection so out of n objects of one object can be selected in C1 ways, 2 objects in C2
n
ways, 3 objects in C3 ways and so on.
Hence, the required number of ways
n
C1  nC2  .....  nCn  2n  1

2nd Floor, Opp. Domino’s, VIP Road, Zirakpur. Ph: 01762-351579, 9815301916, 9878170502
Chase your Dream Permutation and Combination Class : XI 23

This concept has more applications. Let us explore.


(A) Number of Subsets: A set contain ‘n’ element
A = {1,2,3,……n}
If we went to form the subsets of given sets them we can think of any number of elements.
So a subsets can be formed with 0,1,2,….. or with n elements. So required number of subsets,
n n n n n
C0 + C1 + C2 + ….. + Cn = 2

 Again if set A, has 20 elements, number of subsets of A containing at least 12 elements.


20 20 20 20 220  20C10
C11 + C12 + C13 + ….. + C20 =
2
CRACK THIS:
Given, S = {1,2,3,4,5}
Subsets A and B are formed such that n(A  B) = 0 i.e, A and B are dis joint set.
Let us find the number of ways this can be done.
Case – I:
If A has no element then B can have any number of elements, This can be selected in
5 5 5 5 5 5 5
C0 ( C0 + C1 + C2 +….. + C5 )= C0.2 …. (i)
Case II:
If A has 1 element, then B can have maximum number of 4 elements, excluding the element A has .
5 4 4 4 4 4 5 4
This can be selected in C1( C0 + C1 + C2 + C3 + C4) = C12 …. (ii)
Case III:
If A has 2 elements, then B can have maximum number of 3 elements, excluding the element A has. This can
be selected in
5 3 3 3 3 5 3
C2 ( C0 + C1 + C2 + C3) = C2 2 … (iii)
Case IV:
If A has 3 elements,
5 2 2 2 5 2
C3 ( C0 + C1 + C2) = C3.2 …. (iv)
Case V:
If A has 4 elements,
5 4 1 5 1
C4 ( C0 + C1) = C4.2 … (v)

Case VI:
If A has 5 elements
5 5
C5 (°C0) = C5 …. (vi)
Adding all possible cases, total ways of selecting.
5 5 5 4 5 5 5
C0.2 + C1.2 + ….. + C5 = (2+1) = 3 .
Method II
Above problem leads to tedious calculations. Again if we want to create more than 2 disjoint subsets, it will
lead to massive task.
Let no go for tricky and concise concept
Concept:
Say, we have ‘n’ distinct elements each element has two options either it is selected or rejected. Same is
applicable for all the other elements. So required number of ways
n
( 1 + 1 ) (1 + 1) (1 + 1) …………………… (1 + 1) = 2
Selected Re jected

Here we have the same result of total selection. But this concept will give more edge to tougher problems.
Now let no try the above problem.

S = {1,2,3,4,5}

Dustbin
A B

Use
Me

Each element of set S can be thrown to set A, B or to neither A nor B (i.e, dustbin). So in this way A and B will
always be disjoint sets. So, there 3 options (1 to A + 1 to B + 1 to Dustbin) for energy elements.
Hence required number of ways.

2nd Floor, Opp. Domino’s, VIP Road, Zirakpur. Ph: 01762-351579, 9815301916, 9878170502
Chase your Dream Permutation and Combination Class : XI 24

33333=3
5

CRACK THIS
Let S = {1,2,3,4,5,6,7,8}
Number of ways subsets A, B, C are selected such that n(A  B) = 2, n(A  C) = 0, n(C  B) = 0
Concept:
Here A & C are disjoint as well as C and B are disjoint whereas A and B contains exactly two common
elements.
8
First we need to select 2 elements for both A and B in C2 ways. Now for remaining 6 elements each one has
4 options. They can go to A or B or C or nowhere.
Hence required number of ways are
C2  4  4  4  4  4  4 = C2  4
8 8 6

S= {1,2,3,....10}

Dustbin
A B C
a– a– –
Use
b
– b
– – me
– – –
– – –

6.5 Selection of Identical Things:


This is a TRAP. Be Mindful while selecting identical objects.
5
If we want to select 3 letters from the letters of the word AANYA. What will be you answer. C3 ?
Unfortunately No. This will lead to erroneous result.
n
Record it, Note it, Memorize it, Cr is only applicable only when we select ‘r’ distinct objects out of ‘n’
distinct objects.
So what is the solution?
We have to select by forming different cases.
Case 1.
When all there letters are distinct. Note we have 3 A’s one N, one Y. So there are three distinct
3
letters, Hence, selecting 3 distinct letters, C3 ways
Case 2.
When two letters are same, and one is different.
2
Now selecting one pair of A, ‘G way and selecting one distinct letter out of N & Y, C1 ways.
Here required number of ways,
C1  C 1
1 2

Case 3
When all three letters are same we have 1 triplet of A.
1
Hence required number of ways C1
Total selections = C3 + C1  C1 + C1
3 1 2 2

= 1 + 2 + 1 = 4 ways.
We can easily write down,
AAN, ANY, AAY, AAN , AAA
All distinct, 2 same, 1 different, All same
 Let’s navigate to another word ASSASSINATION. In How many ways five letters be selected?
This seems to be challenging one.
ASSASSINATION contains,
4s, 3A, 2I, 2N, 1T, 10
So there are 6 distinct letters.
Case I: When all letters are distinct
6
Number of selections = C5
Case II: 2 letters are same, 4 distinct.
4
For two same letters, we have 4 possibilities. 5, A, I or N. So selections of one pair = C1 ways. And
5
selecting 4 distinct = C4 (Note if pair of 5 is selected then for selecting distinct letters, we have only 5
options left.
Number of selection = C1  C4
4 5

2nd Floor, Opp. Domino’s, VIP Road, Zirakpur. Ph: 01762-351579, 9815301916, 9878170502
Chase your Dream Permutation and Combination Class : XI 25

Case III: 3 letters are same, 2 distinct.


For 3 same letters we have 2 options, S or A
2
So, selections = C1
5
And for distinct letters, 5 options = C2
Number of selection = C1  C2
2 5

Case IV: 4 letters are same, 1 different.


For 4 same letters we have one option.
1
4s, which can be selected in C1 = 1 way.
5
For 1 Different letters, C1 = 5 ways.
Number of Selections = C1  C1 = 5 ways.
1 5

Case V: 3 same, 2 same


2
For 3 letters same of one type we have two options, so selections, C1
3
For 2 letters same of other type we have three options, So selection = C1
Number of selection = C1  C1
2 3

Case VI: 2 same, 2 same, 1 different,


For 2 different pairs, we can 4 options
4
So selections = C2
and for one different letters 4 remaining option
4
So selections = C1
Number of selections = C2  C1
4 4

Summing up all the cases.


C5 + C1 C 4 + C1  C 2 + C1  C1 + C 1  C1 + C2  C1
6 4 5 2 5 1 5 2 3 4 4

Total selection of identical Things (one or more selections)


n
You are well informed now, how to select identical things. Again recap, Cr is not applicable for selecting
identical things.
If there are 5 identical apples number of ways of selecting 3 apples is only one (we can chose any three. Here
5
C3 will give us false result.
Now we are going to discuss all possible ways of selecting identical objects (total selection) which will lead to
tremendous results.
Out of 5 identical apples if we want to look into all the possible selections that will be (1+5). Why?
Here selecting the apple is not important as all of them are identical. But the number of apples are important.
So, there are 5 ways of choosing either one, two, three, four or five apples and one way is for choosing none
of them. Hence the result (1 +5). So they are six ways of selecting 5 identical objects.
Result:
The total number of selecting zero, one or more objects from p identical objects is 1 + p.
Again, the total number of selecting zero, one or more items from p identical objects of one kind, q identical
objects of second kind & identical objects of third kind and n different objects is
(1 + p) (1 + q) (1 + r) (1 1) (1 1) (1 1).....(1 1)
n different objects
n
= (1 + p) (1 +q) (1 + r) 2 .
If at least one object is selected, number of ways will be
n
= (1 + p)(1 + q) (1 + r) 2 – 1

 Number of ways of selecting, one or more fruits out of 3 apples, 2 oranges and one pineapple is
(1 +3) (1 +2) (1 + 1) – 1 = 24 – 1 = 23
We can list all the possible cases for more clarity:
A, O, P, AA, OO, AO, AP, OP, AAA, AAO, AAP, OOA, OOP, OAP, AAAO, AAAP, AAOO, AAOP, OOAP,
AAAOO, AAAOP, AAOOP, AAAOOP
A : Apple, O : Orange, P : Pineapple.

 Number of ways of selecting any number of fruits out of 6 apples, 5 oranges, 4 pineapples, one
watermelon, one banana such that selection has at least 3 apples and 2 oranges.
Now here apples can be selected in 4 ways. Why? (Selection include 3,4,5 or 6 apples).
And similarity oranges can be selected in 4 ways.
Hence required number of selections.
4  4  (1 + 4)(1 + 1) (1 + 1) = 320 ways.
Concept Execution:
A) Number of Divisors: N = 900. How many divisors 900 has ___

2nd Floor, Opp. Domino’s, VIP Road, Zirakpur. Ph: 01762-351579, 9815301916, 9878170502
Chase your Dream Permutation and Combination Class : XI 26

We can write down all of them starting from 1, Easy one, Right ? Not actually.
What if number is bigger?
CONCEPT:
N = 900 = 2  3  5
2

900 is written as a product of prime factors.


We can see if we select any number 2 or 3 or 5, it is the divisor.
Again if 2 and 3 both are selected i.e 6, it is also a divisor. Similarly 3  5 i.e, 45 is also a divisor
2

So consider 2 as apple, 3 as orange and 5 as pineapple.


We have 900 = A  O  P
1 2 1

You got it right. Selecting one apple will give you 2 as divisor, selecting only one orange will give you 3
as divisor, selecting two oranges and one pineapple will give you 3  5 = 45 as divisor.
2

Hence, total number of divisors should be equal to selecting any number of fruits out of one apple, two
oranges, one pineapple i.e,
(1 + 1) (1 + 2) (1 + 1) = 12
So all twelve divisors are
1,2,3,6,9,10,15,18,30,45,450,900
p q r s t
Result: N – 2 3 5 7 . 11 …..
If the number N, is expressed as product of prime factors. Then the total number of divisors are
(1 + p) (1 +q) (1 + r) (1 +s) (1 + t) ….
Note: This result include all the divisors i.e, one and number itself.

 The number of divisors other them 1 and number itself i.e, proper divisors are
(1 + p) (1 + q) (1 + r) (1 + 5) (1 + t) – 2
4 6 3 2
CRACK THIS: N = 2 . 3 . 7 . 11
Now we can easily calculate total divisors or proper divisors of given number. How many of them are
even divisors?
Concept:
Even divisor will generate if at least one 2 is selected so number ways of selecting 2’s, 4 ways (one, two,
three or four 2’s). So total even divisors.
4  (1 + 6) (1 + 3) (1 + 2)

 How many divisors are multiple of 33?


33 is multiple of 3 and 11. So it is mandatory to select at least one 3 and 11.
So, total divisors which are multiple of 33 are
(1 + 4) (6) (1 + 3) (2)
 How many are prime divisors?
It’s an easy one and can be counted effortlessly. You can clearly see, only 2,3,7 and 11 are prime
divisors. Hence, total prime divisors and 4.

(B) Product of two factors:


The numbers of ways of expressing N as a product of two factors.
 1
 (1  p)(1  q)(1  r)(1  s)(1  t), if n is not perfect squa re
2
= 
 1 (1  p)(1  q)(1  r)  (1  s)(1  t)  1, if n is perfect square
 2
Let us make the concept crystal clear.
N = 12 = 2  3
2 1

1
12 can be expressed in (1 2)(1 1)  3
2
No. of ways in which no. can be written as a product of two factors.
Clearly, 12 = 1  12
12 = 2  6 … 3 ways
12 = 3  4
Again if the number is perfect square ,
Let N = 36 = 2  3
2 2

2nd Floor, Opp. Domino’s, VIP Road, Zirakpur. Ph: 01762-351579, 9815301916, 9878170502
Chase your Dream Permutation and Combination Class : XI 27

1
36 can be expressed in (1 2)(1 2)  1  5
2
Clearly,
36 = 1  36,
36 = 2  18, ….(5 ways)
36 = 3  12,
36 = 4  9
36 = 6  6
(C) Product of two factors which are Co-prime to each other
The number of ways in which a composite number N = 1 .2 3 …. k
p q r

(1, 2 …. k are primes) can be resolved into two factors which are co-prime to each other is 2
K–1
,
where K is the number of different prime factor of n.
2 1 1
In N = 60 = 2 . 3 5 , there are only three prime factor 2,3,5.
2 1 3–1 2
So 60 = 2 3 .5 can be resolved into 2 = 2 = 4. Product of two factors which are co-prime
They can be listed as
60 = 1  60
= 4  15
= 3  20
= 5  12
Illustration
The number of divisors of 9600 including 1 and 9600 are
(A) 60 (B) 58 (C) 48 (D) 46
Solution: (C)
Since 9600 = 27  31  5 2 . Hence number of divisors = (7  1)(1  1)( 2  1)  48 .
Illustration
Number of divisors of n  38808 (except 1 and n) is
(A) 70 (B) 68 (C) 72 (D) 74
Solution: (A)
Since 38808 = 8 × 4851 = 8 × 9 × 539 = 8 × 9 × 7 × 7 × 11 = 2 3  3 2  7 2  11
So, number of divisors = (3 + 1) (2 + 1) (2 + 1) (1 + 2) – 2 = 72 – 2 = 70.
Illustration
There are 10 lamps in a hall. Each one of them can be switched on independently. The number of ways
in which the hall can be illuminated is
(A) 10 2 (B) 1023 (C) 2 10 (D) 10 !
Solution: (B)
Number of ways are = 210  1  1023
[– 1 corresponds to none of the lamps is being switched on.]
Illustration
10 different letters of English alphabet are given. Out of these letters, words of 5 letters are formed. How
many words are formed when atleast one letter is repeated
(A) 99748 (B) 98748 (C) 96747 (D) 97147
Solution: (A)
Number of words of 5 letters in which letters have been repeated any times = 10 5
But number of words on taking 5 different letters out of 10 = 10
C5  252

 Required number of words = 10  252 = 99748.


5

Illustration
A man has 10 friends. In how many ways he can invite one or more of them to a party
(A) 10 ! (B) 2 10 (C) 10 !  1 (D) 210  1
Solution: (D)
Required number of friend = 210  1 (Since the case that no friend be invited i.e., 10
C0 is excluded)

2nd Floor, Opp. Domino’s, VIP Road, Zirakpur. Ph: 01762-351579, 9815301916, 9878170502
Chase your Dream Permutation and Combination Class : XI 28

Illustration
Numbers greater than 1000 but not greater than 4000 which can be formed with the digits 0, 1, 2, 3, 4
(repetition of digits is allowed), are
(A) 350 (B) 375 (C) 450 (D) 576
Solution: (B)
Numbers greater than 1000 and less than or equal to 4000 will be of 4 digits and will have either 1
(except 1000) or 2 or 3 in the first place with 0 in each of remaining places.
st
After fixing 1 place, the second place can be filled by any of the 5 numbers. Similarly third place can be
th
filled up in 5 ways and 4 place can be filled up in 5 ways. Thus there will be 5 × 5 × 5 = 125 ways in
which 1 will be in first place but this include 1000 also hence there will be 124 numbers having 1 in the
first place. Similarly 125 for each 2 or 3. One number will be in which 4 in the first place and i.e., 4000.
Hence the required numbers are 124 + 125 + 125 +1 = 375 ways.
Illustration
A student is allowed to select utmost n books from a collection of (2n  1) books. If the total number of
ways in which he can select one book is 63, then the value of n is
(A) 2 (B) 3 (C) 4 (D) None of these
Solution: (B)
Since the student is allowed to select utmost n books out of (2n  1) books. Therefore in order to select
one book he has the choice to select one, two, three,......., n books.
Thus, if T is the total number of ways of selecting one book then T  2n1 C1  2n1 C 2  .....  2n1 Cn  63 .
Again the sum of binomial coefficients
2n 1
C0  2n1 C1  2n1 C 2  ......  2n1 Cn  2n1 Cn1 + 2n 1
Cn 2  .....  2n1 C 2n1  (1  1)2n1  2 2n1
2n 1
or, C0  2(2n1 C1  2n1 C 2  .....  2n1 Cn )  2n1 C 2n1  2 2n1
2 2n1
 1  2(T )  1  2 2n1  1  T   2 2n  1  63  2  2  2  n  3 .
2n 6 2n
2
FINER POINTS
   
Let N  p 1 1 . p 2 2 . p 3 3 ...... p k k , where p1 , p 2 , p 3 ,...... p k are different primes and  1 ,  2 ,  3 ,......,  k are
natural numbers then :
(1) The total number of divisors of N including 1 and N is = ( 1  1) ( 2  1) ( 3  1)....(  k  1)
(2) The total number of divisors of N excluding 1 and N is = ( 1  1) ( 2  1) ( 3  1).....(  k  1)  2
(3) The total number of divisors of N excluding 1 or N is = (1  1)( 2  1)( 3  1).....( k  1)  1
(4) The sum of these divisors is
 ( p10  p 12  p 32  ......  p11 )( p 20  p 12  p 22  ...  p2 2 ).....( pk0  pk1  pk2  ....  pk k )
(5) The number of ways in which N can be resolved as a product of two factors is
1
 2 (1  1)( 2  1)....(  k  1), If N is not a perfect square
1
 [(1  1)( 2  1).....(  k  1)  1], If N is a perfect square
2
(6) The number of ways in which a composite number N can be resolved into two factors which are
relatively prime (or co-prime) to each other is equal to 2 n 1 where n is the number of different
factors in N.
Important Tips
 All the numbers whose last digit is an even number 0, 2, 4, 6 or 8 are divisible by 2.
 All the numbers sum of whose digits are divisible by 3,is divisible by 3 e.g. 534. Sum of the digits is
12, which are divisible by 3, and hence 534 is also divisible by 3.
 All those numbers whose last two-digit number is divisible by 4 are divisible by 4 e.g. 7312, 8936, are
such that 12, 36 are divisible by 4 and hence the given numbers are also divisible by 4.
 All those numbers, which have either 0 or 5 as the last digit, are divisible by 5.
 All those numbers, which are divisible by 2 and 3 simultaneously, are divisible by 6. e.g., 108, 756
etc.
 All those numbers whose last three-digit number is divisible by 8 are divisible by 8.

2nd Floor, Opp. Domino’s, VIP Road, Zirakpur. Ph: 01762-351579, 9815301916, 9878170502
Chase your Dream Permutation and Combination Class : XI 29

 All those numbers sum of whose digit is divisible by 9 are divisible by 9.


All those numbers whose last two digits are divisible by 25 are divisible by 25 e.g., 73125, 2400 etc

DO YOURSELF - 4
Subjective Questions
1. Out of 10 white, 9 black, and 7 red balls, find the number of ways in which selection of one or more
balls can be made (balls of the same color are identical).
2. Find the number of divisors of 720. How many of these are even? Also find the sum of divisors.
Find the number of odd proper divisors of 3  6  21 .
p m n
3.
4. In how many ways the number 7056 can be resolved as a product of 2 factors.
5. Find the number of ways by which 4 prizes can be distributed among 8 students.
6. Find the total number of divisors of 44100.
7. Find the total number of divisors of 103950 which are perfect square.
4 4 4
8. N = 2 6 10 , find the number of divisors of N, which are divisible by 10.
Objective Questions
2 2
1. If a, b, c, d, e are prime integers, then the number of divisors of ab c de excluding 1 as a factor, is
(A) 94 (B) 72 (C) 36 (D) 71
2. The number of proper divisors of 1800 which are also divisible by 10, is
(A) 18 (B) 34 (C) 27 (D) None of these
p m n
3. The number of odd proper divisors of 3 .6 .21 is
(A) (p  1)(m  1)(n  1)  2 (B) (p  m  n  1)(n  1)  1
(C) (p  1)(m  1)(n  1)  1 (D) None of these
p q r
4. The number of proper divisors of 2 .6 .15 is
(A) (p  q  1)(q  r  1) (r  1) (B) (p  q  1)(q  r  1)(r  1)  2
(C) (p  q )(q  r )r  2 (D) None of these
5. The number of even proper divisors of 1008 is
(A) 23 (B) 24 (C) 22 (D) None of these
6. In how many ways can 21 English and 19 Hindi books be placed in a row so that no two Hindi books
are together
(A) 1540 (B) 1450 (C) 1504 (D) 1405
7. A father with 8 children takes them 3 at a time to the Zoological gardens, as often as he can without
taking the same 3 children together more than once. The number of times he will go to the garden is
(A) 336 (B) 112 (C) 56 (D) None of these
8. Ramesh has 6 friends. In how many ways can he invite one or more of them at a dinner
(A) 61 (B) 62 (C) 63 (D) 64
9. Out of 10 white, 9 black and 7 red balls, the number of ways in which selection of one or more balls
can be made, is
(A) 881 (B) 891 (C) 879 (D) 892
10. Out of 6 books, in how many ways can a set of one or more books be chosen
(A) 64 (B) 63 (C) 62 (D) 65
11. In an examination there are three multiple choice questions and each question has 4 choices. Number
of ways in which a student can fail to get all answers correct, is
(A) 11 (B) 12 (C) 27 (D) 63
12. The total number of different combinations of one or more letters which can be made from the letters
of the word ‘MISSISSIPPI’ is
(A) 150 (B) 148 (C) 149 (D) None of these
13. The total number of ways of selecting six coins out of 20 one rupee coins, 10 fifty paise coins and 7
twenty five paise coins is
(A) 28 (B) 56 (C) 37 C6 (D) None of these
14. A fruit basket contains 4 oranges, 5 apples and 6 mangoes. The number of ways person make
selection of fruits from among the fruits in the basket is
(A) 210 (B) 209 (C) 208 (D) None of these

2nd Floor, Opp. Domino’s, VIP Road, Zirakpur. Ph: 01762-351579, 9815301916, 9878170502
Chase your Dream Permutation and Combination Class : XI 30

2 2 2 2
15. The number of ways in which a composite number N = 2 3 5 7 can be resolved into two factors
which are prime to each other is
2 3 4
(A) 2 (B) 2 (C) 2 (D) 2
Answer key
Subjective Answers
 3p nm  1  7n  1 
1. 800 2. 30, 24, 2418 3.    4. 23
 2   2 
7
5. 5 6. 81 7. 4 8. 240
Objective Answers

1 2 3 4 5 6 7 8 9 10 11 12 13 14 15
D A B B A A C C C B D C A B C

6.6 Selection and Arraignment


Finally we have landed up on the concept which will give us edge to solve most of the complicated
problems of this chapter.
Abhi tak, we have dealt the concept of selection i.e, forming a group or a team and arrangement of
objects separately. But now we going to explore both the concepts simultaneously which will open the
window to interesting problems.
Consider the word RANDOM
6
If we want to arrange all letters its 6!. If we want to elect 3 letters its C3
Now if we want to form a three letters word by selecting any three letters and arranging them, we
proceed as.
Selection and Arrangement
C3  3!
6
So,
6
By using C3 we have selected 2 letters and then we have arranged them in 3! Ways.
Although C3  3! = P3 i.e, Pr does both the task simultaneously i.e, it selects r out of n objects and
6 6 n

arrange them as. Well, but when objects are identical it imposes limitations and gives erroneous results.
n
So, we are not going to use Pr.

 How many five letters words can be formed by using letters of the word INTEGRAL such that it contains
exactly 2 vowels. Word INTEGRAL has 3 vowels and 4 consonants. Now selection of vowels and 3
constants can be done in C2  C3 ways.
3 4

So total number of words


vowels consonants
   
3
C 2  4
C 3  5!
Selection and Arrangement

 How many 4 Letter words can be formed by using letters of the word ALGEBRA.
Contain: can be apply  C4  4!
7

Big No, here, word contains two identical letters.


As discussed earlier, we have to from two different cases.
Case I: All distinct letters  First selection then arrangement
C4  4!
6

Case II: 2 same, 2 different


For selection of 2 same, we have only one option,
i.e, one pair of 2A’s
AA___
5
For two different letters, C2 ways.
So total words = 1C1  5 C2  4!
2A 2 different Zi
Arrangement
Selection

 In how many ways 3 Girls & 7 boys be arranged in a row such that Girls are at odd places.
_ ____ _ ____
OEOEOEOEOE
Concept: This seems to be an interesting one. There are only 3 girls but 5 odd places. So we have to
choose any three places where girls can be positioned.

2nd Floor, Opp. Domino’s, VIP Road, Zirakpur. Ph: 01762-351579, 9815301916, 9878170502
Chase your Dream Permutation and Combination Class : XI 31

So selecting 3 places  C3 ways.


5

Now we can arrange these 3 girls in 3! Ways.

Also, remaining 7 places will be occupied by boys in 7 ways


So, required number of ways
C3  3!  7!
5

Concept Equation: Gap Method (Filling Gaps)


Consider the word TRIANGLE, In how many ways word would be arranged such that I and A are never
together?
A simple clear thought with solve this .
Required number of ways.
= Total arrangements – Arrangement where A & I are together
E R AI N T G L
= 8! – 7!  2!
Now the question are, do this method work if out of A, I and E, no two of these are together ?
And also if the restriction of not being together is for more than 3 letters? Actually No
But GAP METHOD is the answer.
Let us understand it.
Leaving apart A, E and I we have 5 letters.
_T_R_N_G_L_
Now there 6 gaps between them and at the end.
The idea is we will select any three spaces and insert A, E and I at these places.
Let’s Go, Selection of 3 spaces  C3 ways
6

Arrangement of A,E & I in then  3! ways


Arrangement of remaining letters  5! Ways
Hence required number of ways
C3  3!  5!
6

 Number of ways of arranging 5 boys and 7 girls in a row such that no two boys are together.

Between seven girls there are 8 gaps again selecting 5 gaps for boys and arrange boys there and also
arranging girls simultaneously, will give required number of ways.
= 8
C5  5!  7!
Selectionof Gaps Arrangement ofBoys Arrangement of Girls

(B) Juggling with numbers.


A four digit number is to be formed which contains exactly two different type of digits.
Case I: Excluding zero
(i) 3 same , 1 different
4!
9
C2  2
C1 
Selecting digits Selectingone which
3!
apper 3 times Arrangement

(ii) 2 same, 2 same


4!
9
C2 
Selecting digits
2!2!
Arrangement

Case II:
Including zero
(i) Zero appears once
3!
9
C1  3
C1 
Selecting other digits Placing zero
3!
Arrangement of other digits

(ii) Zero appearing twice

2nd Floor, Opp. Domino’s, VIP Road, Zirakpur. Ph: 01762-351579, 9815301916, 9878170502
Chase your Dream Permutation and Combination Class : XI 32

2!
9
C1  3
C2   O  O
Selecting other digit Placing zero
2!
Arrangement of other digits

(iii) Zero appearing thrice

9
C1  3
C3  O  O
Selecting other digit Placing zero

Required number of ways


4! 9 4! 9 3! 2!
= 9 C2  2C1   C2   C1  3C1   9C1  3C2   9C1  3C3
3! 2!2! 3! 2!
CRACK THIS:
All possible 4-digit numbers, in each of which the digits occur in non-increasing order (from left to right, e.g.
7664). Find number of such numbers.
Let a1a2a3a4 be number such that a1  a2  a3  a4
Case I : When all digits are unequal.
10
Selections : C4
If 0,1,3,7 are selected, they will be arranged in one way 7310.

Case II: 2 identical, 2 different.


10
So we have to select 3 digits. C3. (say 8,4,0)
3
Again out of these 3, one will repeat, C1(say 0)
Arrangement will be again one way , (8400).
So, required ways, C3  C1
10 3

Case III.
2 identical, 2 identical.
In this case : Select two different number and use them twice.
Selections  C2 (say, 6, 4)
10

Arrangement will be in one way (6644)


10
So required ways, C2
Case IV
3 identical, 1 different
10
We have to select 2 different digits. C2 (say 7,3)
2
Again out of these 2, one will repeat thrice C1 (say 3)
Arrangement will be in one way. (7333)
So, required ways  C2  C1
10 2

Case V
All identical
9
Selections: C1 (excluding 0)
Arrangement: 1 ways
Now combining all the cases,
C4 + C 3  C 1 + C 2 + C 2  C 1 + C 2
10 10 3 10 10 2 a

(C) Number of Functions:


Let A = {1,2,3,4,5} ; B = {1,2,3,4,5,6,7,8}
A function f : A  B is defined
(i) Number of Total Functions: Each element of A can be mapped to any one of element of B. So 1
can be mapped 8 ways. 2 can be mapped again in 8 ways and so on.
Hence required number of mappings. (functions)
=88888=8
5

(ii) Number of One – One Functions: We know in one - one function no two elements of set A should
be mapped to same element of B.
So any 5 elements of B will be mapped to set A. Selecting 5 elements of set B  C5
8

Now, its same as 5 boys of set A will occupy any five chairs of set B.
So total one-one mappings  C5  5!
8

(iii) Number of many one functions


Many- One functions = Total functions – One-One functions
= 8 – C5  5!
5 8

2nd Floor, Opp. Domino’s, VIP Road, Zirakpur. Ph: 01762-351579, 9815301916, 9878170502
Chase your Dream Permutation and Combination Class : XI 33

(iv) Number of functions such that range contains exactly three elements:
Out of 8 given elements in set B, we have to first select 3 elements which should be mapped or will
form the range.
So, selection  C3
8

Let us say, numbers 2,3 and 5 are selected


Now there are two possibilities.
A A

1 1
2 2
3 3
4 4
5 5
6
7
8
(i) Case I: Any three elements of set A will be mapped to one element of B.
Again selecting
5
C3  3
C1  2!
Selection of element Selection of element from Re amining elements to
from set A B to be mapped to 3 mapped, one to each one
elements of set A

(ii) Case II: Five elements to be paired as 2, 2 and 1 and mapped to three elements of set B.
5!
 3!
2!1!
Another approach
= Total functions – Functions which contains only two elements in range – Functions which contains only one
element in range.
8 5 3 5
= C3(3 – C2 (2 – 2) – 3)
Illustration
Seven athletes are participating in a race. In how many ways can the first three athletes win the prizes?
Solution
It is equivalent to filling 3 places (as prizes) with 7 persons. The number of permutations of 7 objects
taken 3 at a time is P3 = 7  6  5 = 210
7

Illustration
How many different signals can be given using any number of flags from 5 flags of different colors?
Solution
The signals can be made by using one or more flags at a time.
The total number of signals when r flags are used at a time from 5 flags is equal to the number of
5
arrangements of 5, taking r at a time, i.e, Pr
Since r can take the values 1,2,3,4,5, by the fundamental principle of addition, the total number of signals
5 5 5 5 5
is P1 + P2 + P3 + P4 + P5
= 5 + (5 + 4) + (5  4  3) + (5  4  3  2) + (5  4  3  2  1)
= 5 + 20 + 60 + 120 + 120= 325
FINER POINTS
(1) Number of permutations of n dissimilar things taken r at a time when p particular things always occur
n p
= Cr  p r!
(2) Number of permutations of n dissimilar things taken r at a time when p particular things never occur
n p
= C r r!
(3) The total number of permutations of n different things taken not more than r at a time, when each
n(n r  1)
thing may be repeated any number of times, is .
n 1
(4) Number of permutations of n different things, taken all at a time, when m specified things always
come together is m ! (n  m  1) !
(5) Number of permutations of n different things, taken all at a time, when m specified things never
come together is n !m !  (n  m  1)!

2nd Floor, Opp. Domino’s, VIP Road, Zirakpur. Ph: 01762-351579, 9815301916, 9878170502
Chase your Dream Permutation and Combination Class : XI 34

Illustration
In a train 5 seats are vacant then how many ways can three passengers sit
(A) 20 (B) 30 (C) 60 (D) 10
Solution: (C)
5! 5! 120
Number of ways are = 5
P3     60 .
(5  3) ! 2! 2
Illustration
How many words comprising of any three letters of the word “UNIVERSAL” can be formed
(A) 504 (B) 405 (C) 540 (D) 450
Solution: (A)
9! 9!
Required numbers of words = 9 P3    504 .
(9  3)! 6 !
Illustration
How many numbers of five digits can be formed from the numbers 2, 0, 4, 3, 8 when repetition of digit is
not allowed
(A) 96 (B) 120 (C) 144 (D) 14
Solution: (A)
Given numbers are 2, 0, 4, 3, 8
Numbers can be formed = {Total – Those beginning with 0}
= {5 ! – 4 !} = 120 – 24 = 96.
Illustration
How many numbers can be made with the help of the digits 0, 1, 2, 3, 4, 5 which are greater than 3000
(repetition is not allowed)
(A) 180 (B) 360 (C) 1380 (D) 1500
Solution: (C)
All the 5 digit numbers and 6 digit numbers are greater than 3000. Therefore number of 5 digit numbers
= 6 P5 5P5  600 .
{Since the case that 0 will be at ten thousand place should be omit}. Similarly number of 6 digit numbers
6 ! – 5 ! = 600.
Now the numbers of 4 digit numbers which are greater than 3000, having 3, 4 or 5 at first place, this can
be done in 3 ways and remaining 3 digit may be filled from remaining 5 digits i.e., required number of 4
digit numbers are 5 P3  3  180 .
Hence total required number of numbers = 600 + 600 + 180 = 1380.
Illustration
All the letters of the word ‘EAMCET’ are arranged in all possible ways. The number of such arrangement
in which two vowels are not adjacent to each other is
(A) 360 (B) 114 (C) 72 (D) 54
Solution: (C)
First we arrange 3 consonants in 3 ! ways and then at four places (two places between them and two
1
places on two sides) 3 vowels can be placed in 4 P3  ways.
2!
1
Hence the required ways = 3 ! × 4 P3   72 .
2!
Illustration
The number of words which can be made out of the letters of the word ‘MOBILE’ when consonants
always occupy odd places is
(A) 20 (B) 36 (C) 30 (D) 720
Solution: (B)
The word ‘MOBILE’ has three even places and three odd places. It has 3 consonants and 3 vowels. In
three odd places we have to fix up 3 consonants which can be done in 3P3 ways. Now remaining three
3
places we have to fix up remaining three places which can be done in P3 ways.
The total number of ways = P3  P3  36 .
3 3

2nd Floor, Opp. Domino’s, VIP Road, Zirakpur. Ph: 01762-351579, 9815301916, 9878170502
Chase your Dream Permutation and Combination Class : XI 35

Illustration
The number of 4 digit number that can be formed from the digits 0, 1, 2, 3, 4, 5, 6, 7 so that each number
contain digit 1 is
(A) 1225 (B) 1252 (C) 1522 (D) 480
Solution: (D)
After fixing 1 at one position out of 4 places, 3 places can be filled by 7 P3 ways. But some numbers
whose fourth digit is zero, so such type of ways = 6 P2
 Total ways = P3  P2  480 .
7 6

Illustration
m men and n women are to be seated in a row, so that no two women sit together. If m  n , then the
number of ways in which they can be seated is
m !(m  1)! m !(m  1)! (m  1)!(m  1)!
(A) (B) (C) (D) None of these
(m  n  1)! (m  n  1)! (m  n  1)!
Solution: (A)
First arrange m men, in a row in m ! ways. Since n < m and no two women can sit together, in any one of
the m ! arrangement , there are (m + 1) places in which n women can be arranged in m 1 Pn ways.
m 1 m !(m  1)!
 By the fundamental theorem, the required number of arrangement = m ! Pn  .
(m  n  1)!
Illustration
If the letters of the word ‘KRISNA’ are arranged in all possible ways and these words are written out as in
a dictionary, then the rank of the word ‘KRISNA’ is
(A) 324 (B) 341 (C) 359 (D) None of these
Solution: (A)
Words starting from A are 5 ! = 120; Words starting from I are 5 ! = 120
Words starting from KA are 4 ! = 24; Words starting from KI are 4 ! = 24
Words starting from KN are 4 ! = 24; Words starting from KRA are 3 ! = 6
Words starting from KRIA are 2 ! = 2; Words starting from KRIN are 2 ! = 2
Words starting from KRIS are 1 ! = 1 Words starting from KRISNA are 1 ! = 1
Hence rank of the word KRISNA is 324
Illustration
We are to form different words with the letters of the word ‘INTEGER’. Let m1 be the number of words in
which I and N are never together, and m2 be the number of words which begin with I and end with R.
Then m1 / m2 is equal to
(A) 30 (B) 60 (C) 90 (D) 180
Solution: (A)
We have 5 letters other than ‘I’ and ‘N’ of which two are identical (E's). We can arrange these letters in a
5!
line in ways. In any such arrangement ‘I’ and ‘N’ can be placed in 6 available gaps in 6 P2 ways, so
2!
5! 6
required number = P2  m1 .
2!
Now, if word start with I and end with R then the remaining letters are 5. So, total number of ways
5! m1 5! 6! 2!
=  m2 .   . .  30 .
2! m2 2! 4! 5!
Illustration
An n digit number is a positive number with exactly n digits. Nine hundred distinct n-digit numbers are to
be formed using only the three digits 2, 5 and 7. The smallest value of n for which this is possible is
(A) 6 (B) 7 (C) 8 (D) 9
Solution: (B)
Since at any place, any of the digits 2, 5 and 7 can be used total number of such positive n-digit numbers
are 3 . Since we have to form 900 distinct numbers, hence 3n  900  n  7 .
n

2nd Floor, Opp. Domino’s, VIP Road, Zirakpur. Ph: 01762-351579, 9815301916, 9878170502
Chase your Dream Permutation and Combination Class : XI 36

Illustration
The number of numbers that can be formed with the help of the digits 1, 2, 3, 4, 3, 2, 1 so that odd digits
always occupy odd places, is
(A) 24 (B) 18 (C) 12 (D) 30
Solution: (B)
4!
The 4 odd digits 1, 3, 3, 1 can be arranged in the 4 odd places, in  6 ways and 3 even digits
2!2!
3!
2, 4, 2 can be arranged in the three even places  3 ways. Hence the required number of ways
2!
= 6 × 3 = 18.
Illustration
How many words of 4 consonants and 3 vowels can be formed from 6 consonants and 5 vowels.
(A) 75000 (B) 756000 (C) 75600 (D) None of these
Solution: (B)
Required number of words = 6 C4 5 C3  7! = 756000
[Selection can be made in 6 C4  5C3 while the 7 letters can be arranged in 7!]
Illustration
To fill 12 vacancies there are 25 candidates of which five are from scheduled caste. If 3 of the vacancies
are reserved for scheduled caste candidates while the rest are open to all, then the number of ways in
which the selection can be made
(A) 5 C3 22 C9 (B) 22 C9 5 C3 (C) 22 C3 5 C3 (D) None of these
Solution: (A)
The selection can be made in 5 C3 22 C9 [since 3 vacancies filled from 5 candidates in 5 C 3 ways and
22
now remaining candidates are 22 and remaining seats are 9, then remaining vacancies filled by C9
ways. Hence total number of ways

DO YOURSELF - 5
Subjective Questions
1. How many numbers greater than 50,000 can be formed by sing the digits 2,5,5,67?
2. How many numbers greater than a million can be formed by using the digits 4,6,0,6,8,4,6?
3. Find the number of words that can be formed by selecting 5 letters of the word INDEPENDENT.
Objective Questions
1. There are three girls in a class of 10 students. The number of different ways in which they can be
seated in a row such that no two of the three girls are together is
10!
(A) 7!  6P3 (B) 7!  8P3 (C) 7! 3! (D)
3!7!
2. The number of ways in which 5 boys and 3 girls can be seated in a row so that each girl in between two
boys
(A) 2880 (B) 1880 (C) 3800 (D) 2800
3. Let S be the set of all functions from the set A to the set A. If n(A)  k then n(S ) is
(C) 2  1
k k k
(A) k ! (B) k (D) 2
4. How many different words, each containing 2 vowels and 3 consonants can be formed with 5 vowels
and 17 consonants?
(A) C2  C3 (B) C2  5! (C) C3  5! (D) C2  C3  5!
5 17 5 17 5 17

5. The number of 6 digit numbers that can be made with the digits 1,2,3 and 4 and having exactly two
pairs of digits, is
(A) 480 (B) 540 (C) 1080 (D) 680
6. The number of five digit even number that can be made with the digits 0,1,2 and 3 is
(A) 384 (B) 192 (C) 768 (D) 576
7. Number of even numbers greater than 300 that can be formed with the digits 1,2,3,4,5 such that no
digit being repeated is
(A) 111 (B) 900 (C) 600 (D) 800

2nd Floor, Opp. Domino’s, VIP Road, Zirakpur. Ph: 01762-351579, 9815301916, 9878170502
Chase your Dream Permutation and Combination Class : XI 37

8. The number of words which can be formed out of the letters of the word PARTICLE, so that vowels
occupy the even place is
(B) C4  C3 (D) 4!  5!
4 3
(A) 7! (C) 180
9. The number of ways in which three girls and ten boys can be seated in two vans, each having
numbered seats, three in the front and four at the back is
14 14 3
(A) C12 (B) P13 (C) 14! (D) 10

Answer key
Subjective Answers
1. 48 2. 360 3. 3320
Objective Answers

1 2 3 4 5 6 7 8 9
B A B D C A A D B

6.7 Divisions and Distribution of distinct object


Division into groups
10 toys are to be divided into two groups: one having 6 toys and other 4 toys.
10 4
Now selecting 6 toys for one group: C6 and from remaining, 4 for other group C4 ways.
So required number of ways
10! 6! 10!
10
C6  6 C4   
6!4! 4!2! 6!4!
Distribution: Now if these toys are to be distributed to two kids. One receiving 6 and other receiving 4,
then required no. of ways in which two groups can be distributed is 2!
10!
So,  2! are the number of ways.
6! 4!
10 Toys

5 3 2
Again if 10 toys are to be distributed to 3 kids such that one receives 5, other 3 and one of them 2.
So applying the same approach, first we from 3 groups by selecting 5,3 and 2 toys.
10!
C5  C3  C 2 =
10 5 2
5! 3! 2!
Now 3 bags of 5,3 and 2 toys respectively can be distributed in 3!
So total way of distributing
10!
 3!
5!3!2!
Concept:
m + n + p+ q

m n p q

Division into groups of m, n, p, q, (no two groups are equal i.e, m  n  p  q),
m+n+p+q n+p+q p+q q
No of ways. Cm Cn CP Cq
(m  n  p  q)!
=
m!n!p!q!
Now if these groups are to be distributed four different individuals.
(m  n  p  q)!
No. of ways  4!
m!n!p!q!

2nd Floor, Opp. Domino’s, VIP Road, Zirakpur. Ph: 01762-351579, 9815301916, 9878170502
Chase your Dream Permutation and Combination Class : XI 38

In how many ways 8 balls of different colors are to distributed to three boxes of different color such that
one receives 4,3 and 1.
8!
Then no. of ways  3!
4! 3! 1!
8!
If all the boxes are of same color ; then is the answer as it is not important which group goes of
4! 3! 1!
which box (all boxes are identical)
Caution
If two group contain equal number of objects.

x,y,z,w x,y,z,w

x,y z,w z,w x,y

As we can be see, the two above divisions are same if both groups contain equal number of objects.
4!
So divisions of x,y,z,w in two group of 2 each will not happen in  6 ways , but in
2!2!
4! 1
  3 ways
2!2! 2!
xyzw

x,y z,w
x,z y,w
z,w y,z
2n! 1
So if 2n objects are divided in two groups of equal size., the  are required number of ways.
n!n! 2!
Again if 3n objects are divided into three groups of equal size, then
3n! 1
 are required number of ways.
n! n! n! 3!
Similarly for r equal group we divide by r!

 Divide 20 problems in 5 equal sections.


20 1
Required number of ways 
4!4!4!4!4! 5!
 Divide 16 different candies in a group of 3,3,2,2,2,2,2.
16! 1 1
Required number of ways =  
3!3!2!2!2!2!2! 2! 5!
Here there are two groups which contains equal number and five groups which contains equal numbers
of candies, so we have to divide by 2! as well as by 5!
Now if these are to be distributed to 7 children required number of ways
16! 1 1
2 5
   7!
(3!) (2!) 2! 5!
CRACK THIS
Number of ways of distributing 6 distinct ball to 4 different boys such that no box is empty.
Red Blue Green Yellow Orange Pink

W X Y Z
Here we can group balls in two ways.
(I) Group of 1,1,1,3

2nd Floor, Opp. Domino’s, VIP Road, Zirakpur. Ph: 01762-351579, 9815301916, 9878170502
Chase your Dream Permutation and Combination Class : XI 39

 6! 1
 1!1!1!3!  3!   4!
 
Division
Distribution

(II) Group of 1,1,2,2


 6! 1 1
 1!1!2!2!  2!  2!   4!
 
Division
Distribution
Total Distribution: In the previous discussion on distribution, we have distributed the objects in specified
way. (i.e, the number of objects for grouping are mentioned).
How many ways are there if we want to distribute any number of balls to any box ?
It seems for total distribution we have to form numerous cases. Don’t you worry. We have a easy way out.
If 6 balls are to be distributed to 4 given boxes. For each ball there are 4 option (it can go to any box).
So required number of distributions:
444444=4
6

This is a big number. It includes all the cases i.e, any box will contain any number of balls and any color.
Hence if three ‘n’ objects which are to be distributed among ‘r’ persons, total number of ways are
r  r  r  r  …..  r = r
n

CRACK THIS:
Number of ways in which 10 different balls are to be distributed in forms boxes red, blue, green and yellow
such that red and green box contains exactly 3 balls each.
10

3 3
Red Green Blue Yellow

First we need to select balls for red and green boxes and distributed them in
10
C3  7 C3 10! 7! 1
 2! =   2!
2! 7! 3!4! 3! 2!
Again remaining 4 balls can be distributed to
Blue and green boxes in 2  2  2  2 = 2
4

10! 7! 1
Hence, required number of ways =   2!  2 4
7!  3!  4!  3! 2!
CRACK THIS
Number of ways in which 20 distinct objects can be placed into two identical boxes no box remains empty.
20
We know, total distribution where boxes are different = 2
20

A B

Excluding two cases when any one of the box is empty  2 – 2


20

Note, here all the cases symmetrical i.e, if in one case, A has 3 objects and B has 17 objects, corresponding
to it there should be one case in which A has 17 objects and B has 3 objects. Since boxes are identical so
both the case will be considered same.
220  2
Hence, required number of ways =  219  1
2
FINER POINTS
Case I : (1) The number of ways in which n different things can be arranged into r different groups is
n  r 1
Pn or n ! n 1Cr 1 according as blank group are or are not admissible.
(2) The number of ways in which n different things can be distributed into r different group is

2nd Floor, Opp. Domino’s, VIP Road, Zirakpur. Ph: 01762-351579, 9815301916, 9878170502
Chase your Dream Permutation and Combination Class : XI 40

r n r C1(r  1)n r C2 (r  2)n  .........  (1)n 1 nCr 1 or Coefficient of x n is n ! (e x  1)r


Here blank groups are not allowed.
(3) Number of ways in which m × n different objects can be distributed equally among n persons
(or numbered groups) = (number of ways of dividing into groups) × (number of groups) ! =
(mn )! n ! (mn )!
 .
(m !)n n ! (m !)n
Case II : (1) The number of ways in which (m  n) different things can be divided into two groups which
m n (m  n)!
contain m and n things respectively is, Cm .n Cn  ,m  n .
m !n!
Corollary: If m  n , then the groups are equal size. Division of these groups can be given by two
types.
Type I : If order of group is not important : The number of ways in which 2n different things can
(2n)!
be divided equally into two groups is
2!(n!)2
Type II : If order of group is important : The number of ways in which 2n different things can be
(2n)! 2n!
divided equally into two distinct groups is 2
 2! 
2!(n!) (n!)2
(2) The number of ways in which (m + n + p) different things can be divided into three groups which
m n  p (m  n  p)!
contain m, n and p things respectively is Cm .n p Cn . p C p  ,m  n  p
m !n ! p !
Corollary: If m  n  p , then the groups are equal size. Division of these groups can be given by
two types.
Type I : If order of group is not important : The number of ways in which 3p different things can
(3 p)!
be divided equally into three groups is
3!(p!)3
Type II : If order of group is important : The number of ways in which 3p different things can be
(3 p)! (3 p)!
divided equally into three distinct groups is 3
t 3!
3!( p!) (p!)3
Note :  If order of group is not important : The number of ways in which mn different things can
mn !
be divided equally into m groups is
(n!)m m !
 If order of group is important: The number of ways in which mn different things can be divided
(mn )! (mn )!
equally into m distinct groups is m
 m!  .
(n!) m! (n!)m
Illustration
In how many ways can 5 prizes be distributed among four students when every student can take one or
more prizes
(A) 1024 (B) 625 (C) 120 (D) 60
Solution: (A)
The required number of ways = 4 5  1024 [since each prize can be distributed by 4 ways]
Illustration
The number of ways in which 9 persons can be divided into three equal groups is
(A) 1680 (B) 840 (C) 560 (D) 280
Solution: (D)
9! 9  8 7  6  5  4
Total ways =  = 280.
(3!)3 3  2  3  2  3  2
Illustration
The number of ways dividing 52 cards amongst four players equally, are
52! 52! 52!
(A) (B) (C) (D) None of these
(13! )4 (13!)2 4 ! (12!) 4 4 !

2nd Floor, Opp. Domino’s, VIP Road, Zirakpur. Ph: 01762-351579, 9815301916, 9878170502
Chase your Dream Permutation and Combination Class : XI 41

Solution: (A)
52 ! 39 ! 26 ! 13 ! 52!
Required number of ways = 52 C13  39C13  26C13  13C13 =    = .
39 !13 ! 26 !13 ! 13 !13 ! 13 ! (13! )4
Illustration
A question paper is divided into two parts A and B and each part contains 5 questions. The number of
ways in which a candidate can answer 6 questions selecting at least two questions from each part is
(A) 80 (B) 100 (C) 200 (D) None of these
Solution: (C)
The number of ways that the candidate may select
2 questions from A and 4 from B = 5 C2 5 C4 ; 3 questions form A and 3 from B = 5 C3 5 C3
4 questions from A and 2 from B = 5 C4  5 C2 . Hence total number of ways are 200.
Illustration
In how many ways can 8 different books be distributed among 3 students if each receives at least 2
books?
Solution
If each receives at least two books, then the division trees would be as shown in fig.
8 8

2 2 4 3 3 2
2
The number of division ways for tree in fig. is 8!(2!) 4!2!. The number of division ways for tree in fig.
2
6.22(ii) is 8!/(3!) 2!2!. The total number of ways of distribution of these groups among 3 students is
 8! 8! 
 2
 2   3!
 (2!) 4!2! (3!) 2!2! 
Illustration
Find the number of ways in which n distinct objects can be kept into two identical boxes so that no box
remains empty.
Solution
Let us consider the boxes to be different.
Each object can be kept in 2 ways (either in box B1 or box B2)
 Total number ways = 2
n

This includes when all the things are in B1 or B2.


 Number of ways = 2 – 2
n

2n  2 n–1
Since the boxes are identical, actual number of ways is =2 –1
2
DO YOURSELF - 6
Subjective Questions
1. Distribute 10 apples
(i) Equally in 5 baskets (ii) Equally among 5 children
2. Find the number of ways in which 5 distinct balls can be put 2 distinct boxes so that none of the boxes
empty.
3. Find the number of ways of distribution of 10 chocolates among three children so that
(i) Every children can get any number of chocolate.
(ii) Every children get at least one chocolate.
Objective Questions
1. The number of ways in which five identical balls can be distributed among ten identical boxes such that
no box contains more than one ball, is
10 ! 10!
(A) 10 ! (B) (C) (D) None of these
5! (5 !)2
2. In how many ways can two balls of the same colour be selected out of 4 black and 3 white balls
(A) 5 (B) 6 (C) 9 (D) 8
3. Ten persons are arranged in a row. The number of ways of selecting four persons so that no two
persons sitting next to each other are selected is
(A) 34 (B) 36 (C) 35 (D) None of these

2nd Floor, Opp. Domino’s, VIP Road, Zirakpur. Ph: 01762-351579, 9815301916, 9878170502
Chase your Dream Permutation and Combination Class : XI 42

4. In a touring cricket team there are 16 players in all including 5 bowlers and 2 wicket-keepers. How
many teams of 11 players from these, can be chosen, so as to include three bowlers and one wicket-
keeper
(A) 650 (B) 720 (C) 750 (D) 800
5. A total number of words which can be formed out of the letters a, b, c, d, e, f taken 3 together such that
each word contains at least one vowel, is
(A) 72 (B) 48 (C) 96 (D) None of these
6. Out of 6 boys and 4 girls, a group of 7 is to be formed. In how many ways can this be done if the group
is to have a majority of boys
(A) 120 (B) 90 (C) 100 (D) 80

7. Let A be a set containing 10 distinct elements. Then the total number of distinct functions from A to A, is
(D) 2  1
10 10 10
(A) 10 ! (B) 10 (C) 2
8. A lady gives a dinner party for six guests. The number of ways in which they may be selected from
among ten friends, if two of the friends will not attend the party together is
(A) 112 (B) 140 (C) 164 (D) None of these
9. The number of ways in which mn students can be distributed equally among n sections is
(mn )! mn mn
(A) (mn )n (B) (C) (D)
(m !)n m! m!n !
10. There are 3 candidates for a post and one is to be selected by the votes of 7 men. The number of ways
in which votes can be given is
7
(A) 73 (B) 3 (C) 7 C 3 (D) None of these
11. In how many ways can 10 different balls be divided between two boys, one receiving two and the other
eight balls
(A) 45 (B) 75 (C) 90 (D) None of these
12. The number of ways in which six different prizes can be distributed among three children each
receiving at least one prize is
(A) 270 (B) 540 (C) 1080 (D) 2160
13. The number of ways in which a pack of 52 cards can be divided equally among four players in orders is
52! 52! 52! 52!
(A) (B) (C) (D)
(13!) 4 4
(13!) 4! (13!)  4
4
13! 4!
14. The number of ways in which a pack of 52 cards can be divided equally into four groups, is
52! 52! 52! 52!
(A) (B) (C) (D)
(13!) 4 4
(13!) 4! (13!)  4
4
13! 4!
15. The total number of ways of dividing 15 distinct things into groups of 8,4,3 respectively, is
15! 15! 15! 15!
(A) (B) (C) (D)
8! 9!(3!)2 8!4!3! 8!4! (8!) 2
4! 3!
16. The number of ways of distributing 12 different objects among three persons such that one particular
person is always getting 6 objects and the remaining two persons are getting three objects each , is
12! 12! 12! (12)!
(A) (B) (C) (D)
6!(3!)2 2! 6!(3!)2 (6!)2 (3!)2 (6!)2

Answer key
Subjective Answers
1. (i) 945, (ii) 113400 2. 30 3. (i) 66, (ii) 36
Objective Answers

1 2 3 4 5 6 7 8 9 10 11 12 13 14 15 16
C C C B C C B B B B C B A B B B

2nd Floor, Opp. Domino’s, VIP Road, Zirakpur. Ph: 01762-351579, 9815301916, 9878170502
Chase your Dream Permutation and Combination Class : XI 43

6.8 Divisions & Distribution: Identical objects


Division into Groups
How many ways 10 identical apples be divided into two groups containing 6 and 4 objects.
Do we need to select 6 apples for first group  i.e, C6 .
10

10

6 4
Big No. B’coz objects are identical, there is no need to select. So how to divide them. No prizes for
guessing only one way. Just keep 6 on one side and 4 on other side.

Again if apples are to be divided in three group of 5,4,1, three will be only one way.

So division of any number of objects and to any numbers of groups, there will be again only on way.
Distribution: Now if these 5,4 and one apples are to be distributed to three persons. We should do in 3!
Ways.
Hence required number of ways.
1  3!
Division
Distribution

CRACK THIS: Divide 5 identical balls in three identical boxes in a group of 2, 2 and 1.

2 2 1 Only one way


Again divide 5 identical balls in their identical boxes such that no box is empty.

2 2 1
1 1 3 }2 ways
Now if boxes are different,
X Y Z
2 2 1  3!
 3 Ways
2!
1 1 3  3!
 3 Ways
2!
Hence required number of ways = 3 + 3 = 6

Total Distribution
In the previous discussion we have divided and distributed identical objects in a specified nos. (number of
objects for each group were mentioned).
Now, we are looking at total distribution i.e, any person can receive any number of objects. This is very crucial
and tricky concept.
Let us have 5 toy cars which are to be distributed among 3 kinds. We can write this problems in equation
form.
x + y + z = 5, x, y, z  0
Here x, y, z are integers and represents number of toys to first, second and third kid. Let us explore this
concept.

2nd Floor, Opp. Domino’s, VIP Road, Zirakpur. Ph: 01762-351579, 9815301916, 9878170502
Chase your Dream Permutation and Combination Class : XI 44

Concept:

Here fig shows 5 cards and two dots. Let arrange 5 cars and 2 dots. One of the way will be

Before first dot we have one car in between 2 dots we have two cars.
And after 2 dot we have again two cases.
So, distribution will be 1, 2 and 2
So total ways are equal to arranging 5 cars and 2 dots.
7! 67
i.e,   21
5!2! 2
We can write the all possible ways for more clarity.

X Y Z X Y Z X Y Z
5 0 0 0 4 1 0 0 5
4 1 0 0 3 2 1 0 4
3 2 0 0 2 3 2 0 3
2 3 0 0 1 4 3 0 2
1 4 0 1 3 1 4 0 1
1 1 3 2 1 2 3 1 1
1 2 2 2 2 1

Required number of ways are 21


Now we can generate the result for n identical objects which are to be distributed to r person.

(r – 1)dots
n–objects
So the number of ways are same as arranging n-objects and r – 1 dots in a row,
(n  r  1)! n r 1
 Cr 1
n!(r  1)!
Also the number of ways are same as number of non-negative integral solutions of
x1 + x2 + …… + xq = n ; x1, x2 , ….. xr  0 …. (i)
If we need to distribute 20 identical toys to 5 kids such that anyone of them receives any number of
objects then it can be represented in form of equation as number of non-negative integral solutions of
x1 + x2 + x3 + x4 + x5 = 20 ; x1, x2, x3, x4 , x5  0
Here, n = 20, r = 5, and number of solutions is
n+r–1 20+5-1 24
Cr–1 i.e, C5–1 = C4
What if we want to find number of integral solutions of
x + y + z + w = 15
Such that x  1, y  2, z  0, w  3
Let x = 1 + t1, y = 2 + t2, z = t3, w = 3 + t4
Equation changes to
t1 + t2 + t3 + t4 = 9, t1, t2, t3, t4  0
Now the condition is same as (i) and result is very well applicable.
9+4–1 12
Hence the required number of ways are C4-1 or C3

CRACK THIS
In how many ways 3 girls and 12 boys can sit in a row such that between any two girls at least 3 boys are
seated ?
_ _ _ _ _ _ G1 _ _ _ _ _ G2 _ _ _ _ _ G3 _ _ _ _ _
x y z w
As shown in fig. x, y, z and w are number of boys between girls and at ends.

2nd Floor, Opp. Domino’s, VIP Road, Zirakpur. Ph: 01762-351579, 9815301916, 9878170502
Chase your Dream Permutation and Combination Class : XI 45

Now required number of ways are same as number of solutions of


x + y + z + w = 12, x 0, y  2, z  2, w  0
Following the last approach number of solutions are
8+4-1 11
C4-1 = C3
C3  3!  12!
11
But, again girls and boys are also to be arranged so required number of ways are
CRACK THIS
Number of odd integral solution of the equation a + b + c + d = 30
Here, a, b, c , d are odd , so we can substitute
a = 2t1 + 1, b = 2t2 + 1, c = 2t3 + 1, d = 2t4 + 1
We have
2t1 + 1 + 2t2 + 1 + 2t3 + 1 + 2t4 + 1 = 30 ; t1, t2, t3, t4  0
t1 + t2 + t3 + t4 = 13
Number of solutions 
13+4–1 16
C4–1 = C3
CRACK THIS
Number of non-negative integral solution of x + y + z = 30, x  y  z
We have, total solutions of x + y + z = 30, x, y, z  0 are
30+3–1 32
C3–1 = C2
Required number of solutions =
Total solutions – Solutions when any two are equal – Solutions when all equal

When all three are equal, x = y = z = 10, one solution


When two are equal, Let x = y  z ; 2x + z = 10
Number of solutions can be easily counted, x = 0, 1,2,3,4, 5 solutions..
Again when x  y = z and x = z  y also has 5 sols.
Hence, required number of solutions= C2 – 3  5 – 1
32

CRACK THIS
Number of non-negative integral solution of
x + y + z  15
The sum of x, y, z is less than of equal to 1s. So if we a add a variable to it say t which will exactly gets
added to x + y + z, to make the sum 15
x + y + z + t = 15, x, y, z, t  0
15+4–1 18
Now the number of solution of this equation is, C4–1 = C3
CRACK THIS
Number of non-negative integral solution of 10  x + y + z  30
This is a real challenger. Can you crack this?
Here we know how to find solutions of x + y + z  30 from precious approach yes by adding variable t,
30+4–1 33
x+ y + z + t = 30 which are equal to C4–1 = C3 , but it includes all the solutions less than 10.
So we have to exclude all the solutions of x + y + z  9.
Now number of solutions of x +y + z  9 or x + y + z + t = 9 are
9+4-1 12
C4–1 = C3
30 12
So, required number of solutions C3 – C3

6.9 Multinomial Theorem


We are now well acquainted with the distribution of ‘n’ identical objects to ‘r’ persons or number of non-
negative integral solutions of x1 + x2 + …. + xr = x,
n+r–1
which is equal to Cr–1
But this result has a Big Limitation.
Limitation: If any of the variable x1, x2 ….. or xn does not take value upto ‘n’ then the result is not
applicable.
Let us get more clarity with an example.
A dice is rolled three times. In how many ways we can get a sum of 10.
Here, x + y + z = 10 ; 1  x, y , z  6
As we can clearly see, x, y, z can get the maximum value upto 6 and the total sum is 10. So we can’t
apply the result. We have another approach to tackle these problems, multinomial theorem.
1 dice can take value 1,2,….,6 which we write as
1 2 3 4 5 6
d +d +d +d +d +d
Here power represents value which the dice turns up. Similarly second & third dice can take values from
1 to 6.
Now, we multiply the three series.

2nd Floor, Opp. Domino’s, VIP Road, Zirakpur. Ph: 01762-351579, 9815301916, 9878170502
Chase your Dream Permutation and Combination Class : XI 46

1 2 3 4 5 6 1 2 3 4 5 6 1 2 3 4 5 6
(d +d +d +d +d +d ) (d +d +d +d +d +d ) (d +d +d +d +d +d )

10
Here, the number of times d appears will be same as number of ways the sum will be 10.
1 3 6 10
As shown in fig. Where d , d and d multiply we get one d and it represents that first dice shows one,
second dice show 3 and third dice shows 6.
d 
3
 d2  ......  d6
1 10
So, in the expansion of , the coefficient of d will be equal to required number of
cases.
10 2 6 3
Coefficient of d in (d + d + …. + d )
10 3 5 3
= Coefficient of d in d (1 + d+ ….. +d )
3
 1  d6  7
= Coefficient of d in  
 1 d 
7 6 3 3
= Coefficient of d in (1 – d ) (1 – d)
7 3 6 3 4 2 11 7
= Coefficient of d in (1 – C1 d + ….. )(1 + C1 d + C2 d + ….. + C7 d + ….. )
= C7 – C1  C1
11 3 3

So, in order to find the number of non-negative integral solutions of equation


x1 + x2 + …… + xr = n, 0  x1x2 ….. x4  n
n
The required number of solution is equal to is the coefficient of x in
0 1 n 0 1 n 0 1 n
(x + x + …… + x ) (x + x + …. +x ) …. (x + x + ….. x ) (r-bracket)
n 2 n r
= coefficient of x in (1 + x + x + …. + x )
r
n  1  x n 1 
= coefficient of x in  
 1 x 
n n+1 +r –r
= coefficient of x in (1 – x ) (1 – x)
n r n+1 r r+1 2
= coefficient of x in (1 – C1 x + …… )(1 + C1x + C2x + ….. )
n r r+1 2 n+r–1
= coefficient of x in (1 + C1x + C2 x + …… ) = Cr–1
CRACK THIS:
In how ways can 14 identical candies can be distributed among 3 girls and 3 boys such that girls get
equal number of candies, and two particular boys equal candies?
Let the girls get m, m, m candies and two particular boys get n, n candies and third boy get  candies.
So, we have m + m + m + n + n +  = 14 or 3m + 2n +  = 14; m, n,   1
14
The number of solutions is equal to the coefficient of x in
3 6 9 12 2 4 14 1 2 14
(x + x + x + x ) (x + x + …. + x ) (x + x + …. x )
8 3 6 2 8 7 8
= coefficient of x in (1 + x + x ) (1 + x + ……. x )(1 + x + x )
= coefficient of x in (1 + x + x + x + x + 2x + x + 2x )  (1 + x+ x + ……. + x )
8 2 3 4 5 6 7 8 2 8

= 1 + 1 + 1 + 1 + 1 + 2 + 1 + 2 = 10 ways.
DO YOURSELF - 7
Subjective Questions
1. Find the number of ways in which 16 constables can be assigned to patrol 8 villages, 2 for each.
2. Find the number of ways of selecting 10 balls out of an unlimited number of identical white, red, and
blue balls.
3. In how many ways, two different natural numbers can be selected, which are less than or equal to 100
and differ by almost 10.
4. Find the number of positive integral solutions of xyz = 21600.
5. Find the number of positive integral solutions satisfying the equation (x 1 + x2 + x3)(y1 + y2) = 77.
6. If x1 x2 x3 = 70, then find the number of positive integral solution.
7. The number of integral solution of x1 + x2 + x3 + x4 = 25. Such that x1  1, x2  2, x3  3, x4  4.
8. Find the number of way to distribute 15 identical things among four persons if each gets at least two
things.
9. Find the number of ways of assigning 18 identical paintings to different dormitories.
Objective Questions
1. The number of ways in which thirty five apples can be distributed among 3 boys so that each can have
any number of apples, is
(A) 1332 (B) 666 (C) 333 (D) None of these

2nd Floor, Opp. Domino’s, VIP Road, Zirakpur. Ph: 01762-351579, 9815301916, 9878170502
Chase your Dream Permutation and Combination Class : XI 47

2. The total number of integral solutions of the equation x + y + z + t = 29 where x  1, y  2, z  3 and t  0 is


26 26 22 29
(A) C4 (B) C3 (C) C3 (D) C3
3. In how many ways can four persons, each throwing a die once, make a sum of 6?
(A) 12 (B) 8 (C) 16 (D) 10
4. The number of integer solution for the equation x + y + z + t = 20, where x, y, z, t are all  –1, is
20 23 27 27
(A) C4 (B) C3 (C) C4 (D) C3
5. The number of non-negative integral solutions of x + y + z  n, where n  N is
n+3 n+4 n+5 n+2
(A) C3 (B) C4 (C) C5 (D) C3
6. The total number of positive integral solutions for (x, y, z) such that xyz = 24 is
(A) 36 (B) 90 (C) 120 (D) 30
7. The number of points in space, whose each co-ordinate is a negative integer such that x + y + z + 12 = 0
(A) 385 (B) 55 (C) 110 (D) 120
8. The number of ways in which an examiner can assign 30 marks to 8 questions, giving not less than 2
marks to any question, is
21 21 21 20
(A) C7 (B) C8 (C) C9 (D) C7
Answer key
Subjective Answers
32
6. 27 7. 816 8. 120 9. C14
Objective Answers
1 2 3 4 5 6 7 8
B B D D A D B A
6.10 Circular Permutations
So far we have been considering the arrangements of objects in a line. Such permutations are known as
linear permutations.
Instead of arranging the objects in a line, if we arrange them in the form of a circle, we call them, circular
permutations.
In circular permutations, what really matters is the position of an object relative to the others.
Thus, in circular permutations, we fix the position of the one of the objects and then arrange the other
objects in all possible ways.
There are two types of circular permutations :
(i) The circular permutations in which clockwise and the anticlockwise arrangements give rise to
different permutations, e.g. Seating arrangements of persons round a table.
(ii) The circular permutations in which clockwise and the anticlockwise arrangements give rise to same
permutations, e.g. arranging some beads to form a necklace.
Look at the circular permutations, given below :

A A

D B B D
C C

Suppose A, B, C, D are the four beads forming a necklace. They have been arranged in clockwise and
anticlockwise directions in the first and second arrangements respectively.
Now, if the necklace in the first arrangement be given a turn, from clockwise to anticlockwise, we obtain
the second arrangement. Thus, there is no difference between the above two arrangements.
(1) Difference between clockwise and anticlockwise arrangement : If anticlockwise and clockwise order
of arrangement are not distinct e.g., arrangement of beads in a necklace, arrangement of flowers in
(n  1)!
garland etc. then the number of circular permutations of n distinct items is
2
(2) Theorem on circular permutations
Theorem 1 : The number of circular permutations of n different objects is (n  1) !
Theorem 2 : The number of ways in which n persons can be seated round a table is (n  1)!
1
Theorem 3: The number of ways in which n different beads can be arranged to form a necklace, is (n  1)!
2

2nd Floor, Opp. Domino’s, VIP Road, Zirakpur. Ph: 01762-351579, 9815301916, 9878170502
Chase your Dream Permutation and Combination Class : XI 48

FINER POINTS
 When the positions are numbered, circular arrangement is treated as a linear arrangement.
 In a linear arrangement, it does not make difference whether the positions are numbered or not.
Illustration
In how many ways a garland can be made from exactly 10 flowers
9!
(A) 10 ! (B) 9 ! (C) 2 (9!) (D)
2
Solution: (D)
1 1
A garland can be made from 10 flowers in (9!) ways [  n flower's garland can be made in (n  1)!
2 2
ways]
Illustration
In how many ways can 5 boys and 5 girls sit in a circle so that no boys sit together
5!  5!
(A) 5! × 5! (B) 4! × 5 ! (C) (D) None of these
2
Solution: (B)
Since total number of ways in which boys can occupy any place is (5  1)!  4! and the 5 girls can be sit
accordingly in 5! ways. Hence required number of ways are 4 ! × 5 !.
Illustration
The number of ways in which 5 beads of different colours form a necklace is
(A) 12 (B) 24 (C) 120 (D) 60
Solution: (A)
The number of ways in which 5 beads of different colours can be arranged in a circle to form a necklace
are = (5  1)!  4 ! .
But the clockwise and anticlockwise arrangement are not different (because when the necklace is turned
1
over one gives rise to another). Hence the total number of ways of arranging the beads = (4 !)  12 .
2
Illustration
The number of ways in which 5 male and 2 female members of a committee can be seated around a
round table so that the two female are not seated together is
(A) 480 (B) 600 (C) 720 (D) 840
Solution: (A)
Fix up a male and the remaining 4 male can be seated in 4! ways. Now no two female are to sit together
and as such the 2 female are to be arranged in five empty seats between two consecutive male and
number of arrangement will be 5 P2 . Hence by fundamental theorem the total number of ways is =
4!  5 P2  24  20  480 ways.
Illustration
Five boys and five girls sit alternately around a round table. In how many ways can this be done?
Solution
Five boys can be arranged in a circle in 4! Ways.
b5 b 1

b4

b2
b3
After that girls can be arranged in the 5 gaps shown as ‘’ in 5! Ways. Hence, total number of ways is
4!  5! = 2880

2nd Floor, Opp. Domino’s, VIP Road, Zirakpur. Ph: 01762-351579, 9815301916, 9878170502
Chase your Dream Permutation and Combination Class : XI 49

Illustration
A person invites a group of 10 friends at dinner and sits
(i) 5 on a round table and 5 more on another round table,
(ii) 4 on one round table and 6 on the other round table.
Find the number of ways in each case in which he can arrange the guests.
Solution
10
(i) The number of ways of selection of 5 friends for the first table is C5. Remaining 5 friends are left for
the second table.
The total number of permutations of 5 guests on each tale is 4!. Hence, the total number of
arrangements is C5  4!  4! = 10!/(5!  5!)4!  4! = 10!/25
10
10
(ii) The number of ways of selection of 6 guests is C6 . The number of ways of permutations of 6
guests on round table is 5!. The number of permutation of 4 guests on round table is 3!.
Illustration
Find the number of ways in which 10 different diamonds can be arranged to make a necklace.
Solution
Since diamonds do not have natural order of left and right so clockwise and anticlockwise arrangements
are taken as identical. Therefore, the number of arrangements of 10 different diamonds to make a
necklace is 1/2  9 = 181440.
Illustration
Find the number of ways in which six persons can be seated at a round table, so that all shall not have
the same neighbours in any two arrangements.
Solution
In this case, anticlockwise and clockwise arrangements are the same.
Hence, the number of ways of arrangements is 5!/2 = 60.
6.11 Exponent of Prime p in n ! .
Let p be a prime number and n be a positive integer. Then the last integer amongst 1, 2, 3, .......(n – 1), n
n  n  n
which is divisible by p is   p , where   denote the greatest integer less than or equal to .
p p p
10  12  15 
For example:    3 ,    2,    5 etc.
3 5 3
Let E p (n) denotes the exponent of the prime p in the positive integer n. Then,

 n   n   n 
E p (n !)  E p (1 .2 .3 .......... (n  1) n) = E p  p. 2 p. 3 p.......   p  =    E p  1 . 2 . 3 ......   
 p  p   p 
[ Remaining integers between 1 and n are not divisible by p]
n 
Now the last integer amongst 1, 2, 3,.....   which is divisible by p is
p
n / p   n   n   n  
 p    2    p   Ep  p, 2 p, 3 p....  2  p  because the remaining natural numbers from 1 to
  p     p  
n  n   n    n 
p     E  1 . 2 . 3 ......  2  
p are not divisible by p = p
   
2
  p   p 
n   n   n   n 
Similarly we get E p (n !)      2 
 3   .....  S 
p p  p  p 
where S is the largest natural number. Such that p S  n  p S 1 .
Illustration
50 50
Find the highest power of 3 in C10 & P10 .
Solution
50!  50   50   50   50 
 H 3 (50!)             = 16 + 5 + 1 + 0 = 22
50
We have C10 =
10!4!  3   9   27   81

2nd Floor, Opp. Domino’s, VIP Road, Zirakpur. Ph: 01762-351579, 9815301916, 9878170502
Chase your Dream Permutation and Combination Class : XI 50

 40   40   40   40 
H 3 (40!)             = 13 + 4 + 1 + 0 = 18
 3   9   27   81 
 10   10   10 
H 3 (10!)          = 3 + 1 +0 = 4  H3( C10) = H3(50!) – H3(40!) – H3(10!) = 22 – 18 – 4 = 0
50

     
3 9 27
50
Also H3( P10) = H3(50!) – H3(40!) = 22 – 18 = 4

DO YOURSELF – 8
Subjective Questions
1. In how many ways can 3 ladies and 3 gentlemen be seated around a round table so that any two and
only two of the ladies sit together?
2. Find the number of ways in which 6 men and 5 women can dine at a round table if no two women are
to sit together.
3. Find the number of ways in which 8 different flowers can be strung to form a garland so that four
particular flowers are never separated.
4. In how many ways, 20 guests and 1 host can be arranged around a circular table so that two particular
guests are on either side of the host?
5. Find the number of garlands made by 6 distinct beads.
6. The exponent of 2 in 30  29  28  ……  12  11 is ______
30
7. The number of zeroes in the end of P10.
Objective Questions
1. If eleven members of a committee sit at a round table so that the president and secretary always sit
together, then the number of arrangement is
(A) 10 ! × 2 (B) 10 ! (C) 9 ! × 2 (D) None of these
2. In how many ways can 5 keys be put in a ring
4! 5!
(A) (B) (C) 4 ! (D) 5!
2 2
3. In how many ways can 12 gentlemen sit around a round table so that three specified gentlemen are
always together
(A) 9 ! (B) 10 ! (C) 3 ! 10! (D) 3 ! 9 !
4. n gentlemen can be made to sit on a round table in
1 1
(A) (n  1)! ways (B) (n  1)! ways (C) (n  1)! ways (D) (n  1)!ways
2 2
5. In how many ways 7 men and 7 women can be seated around a round table such that no two women
can sit together
(A) (7!)2 (B) 7! 6! (C) (6!)2 (D) 7 !
6. The number of circular permutations of n different objects is
(A) n ! (B) n (C) (n  2)! (D) (n  1)!
7. The exponent of 2 in 100!
(A) 47 (B) 97 (C) 98 (D) 50
8. The number of zeroes at the end of 100!
(A) 22 (B) 23 (C) 24 (D) 25
9. The exponent of 7 in 100! Is
(A) 14 (B) 15 (C) 16 (D) 17
40
10. The exponent of 3 in C10
(A) 2 (B) 0 (C) 3 (D) 4
Answer key
Subjective Answers
4. 2.18! 5. 60 6. 18 7. 3
Objective Answers

1 2 3 4 5 6 7 8 9 10
C A D B B D B C C B

2nd Floor, Opp. Domino’s, VIP Road, Zirakpur. Ph: 01762-351579, 9815301916, 9878170502
Chase your Dream Permutation and Combination Class : XI 51

Example
How many different numbers of 4 digits can be formed from the digits 0,1,2,…..9 if repetition is
(i) allowed (ii) not allowed
Solution
(i) Repetition is allowed.
The first placed is filled by any number from 1 to 9 as 0 cannot occur at the first place and each of the
remaining 3 places can be filled by any one of the digits from 0,1, …., 9 , i.e, in 10 different ways.

9 10 10 10
Hence, the total number of four –digit numbers that can be formed is 9  10  10  10 = 9  10
4

(ii) Repetition is not allowed.


The total number is 9  9  8  7 = 4536. Hence, the total number of four-digit numbers that can be
formed is 9  P3.
9

9
9 P3

Example
How many our-digit numbers can be formed by using the digits 1,2,3,4,5,6,7 if at least one digit is
repeated.
Solution
4
Numbers that can be formed when repetition of digits is allowed are 7 .
7
Numbers that can be formed when all the digits are distinct or when repletion is not allowed are P4.
4 7
Therefore, numbers that can be formed when at least one digit is repeated are 7 – P4.

Example
Find the number of seven letters words that can be formed by using the letters of the word Success so
that the two C are together but no two S are together.
Solution
Considering CC as single object, U, CC, E can be arranged in 3! Ways
 U  CC  E 
Now the three S are to be placed in the four available places ()
Hence, required number of ways = 3!  C3 = 24.
4

Example
There are n married couples at a party. Each person shakes hand with every person other than her or his
spouse. Find the total number of hands shakes.
Solution
2n
Total hand shakes possible = C2.
This will include n hand shakes in which a person shakes hand with her or his spouse.
 Required number = C2 – n = 2n(n – 1)
2n

Example
Find the number of ways in which A A A B B B can be placed in the squares of Fig. as shown, so that no
row remains empty.

2nd Floor, Opp. Domino’s, VIP Road, Zirakpur. Ph: 01762-351579, 9815301916, 9878170502
Chase your Dream Permutation and Combination Class : XI 52

Solution
nd th st rd th
2 and 4 row block has to selected. One row out of 1 , 3 , and 5 will contain two letters and remaining
one-one letter.
3
Selection of one row that contain two letters is done in C1 ways.
3
Now two block can be selected from this row in C2 ways.
3
From each of remaining two rows, one blocks can be selected in C1 ways.
3 3 3 3
Hence, total selections = C1 . C2 . C1. C1 = 81
6!
Number of ways of filling A, A, A, B, B, B = 81 
3!3!
= 81  20 = 1620
Example
Nishi has 5 coins, each of the different denomination. Find the number different sums of money she can
form.
Solution
Number of different sums of money she can form is equal to number of ways she select one or more
coins. Therefore,
5 5 5 5 5
Required number of ways = C1 + C2 + C3 + C4 + C5
5
= 2 – 1 = 31
Example
3 5 7 9
Find the number of divisors of the number N = 2 .3 .5 .7 which are perfect squares.
Solution
Since the divisor is a perfect square, each prime factor must occur even number of times.
0 2
2 can be taken in 2 ways (2 or 2 )
0 2 4
3 can be taken in 3 ways (3 or 3 or 3 )
0 2 4 6 0 2 4 6
Similarly, 5 can be taken in 4 ways (5 or 5 or 5 or 5 ) and 7 can be taken in 5 ways (7 or 7 or 7 or 7
8
or 7 )
Hence, total divisors which are perfect squares = 2  3 4  5 = 120
Example
Find the number of ways in which the number 300300 can be split into two factors which are relatively
prime.
Solution
3 1 2 1 1 1
300300 = 2 3 5 7 11 13
Now we have to make factors which are relative prime.
 2 , 3 , 5 , 7 , 11 , 13 should behave as single identities.
2 1 2 1 1 1

So number of divisors (1 + 1)(1 + 1)(1 + 1)(1 + 1)(1 + 1)(1 + 1)


6
= 2 = 64
64
Number of ways of splitting into 2 factors =  32
2
Example
Find the number of non-negative integral solutions of the equation x + y + z + 2w = 20.
Solution
Let w = 0. Then, the equation reduces to x + y + z = 20.
20+3–1 22
Number of non-negative integral solutions is C3–1 = C2 .
18+3–
If w = 1, then the equation reduces to x + y + z = 18. Number of non-negative integral solutions is
1 20
C3–1 = C2 .
Similarly, we have w = 2,3, … , 10
Therefore, the total number of solutions is
22 20 18 6 4
C3 + C3 + C3 + …. + C3 + C3 .
Example
Find the total number of positive integral solutions for (x, y, z) such that xyz = 24. Also find out the total
number of integral solutions.
Solution
24  2  3
3

Now, consider three boxes x, y, z. Three can be put in any of the three boxes.
3+3–1 5
Also, 2,2,2 can be distributed in the three boxes in C3–1 = C2 ways. Hence, the total number of
positive integral solutions is equal to the number of distributions which is given by 3  C2 = 30.
5

2nd Floor, Opp. Domino’s, VIP Road, Zirakpur. Ph: 01762-351579, 9815301916, 9878170502
Chase your Dream Permutation and Combination Class : XI 53

Example
2 5 1
Consider the equation   , where x, y  N. Find the number of solutions of the equation.
x y 3
Solution
2 5 1
Given equation is  
x y 3
or 6y + 15x = xy
or (x –6)(y – 15) = 90 = 2  3  5
2

Consider (x – 6) and (y – 15) as boxes,


Number of ways 2 can be distributed = 2
Number of ways 5 can be distributed = 2
Number of ways (3, 3) can be distributed = 3
 Total number of solutions = 2  2  3 = 12
Note: If any box remains empty, say x, then x = 1. To find integral solutions where negative integers are
also allowed.
Any two of the factors in each factorization may be negative. Therefore, the number of ways to associate
negative sign in each case is C2 = 3. Hence, the total number of integral solutions is 30+ 3  30 = 120.
3

Example
In how many ways can 10 persons take seats in a row of 24 fixed seats so that no two persons take
consecutive seats.
Solution
As no two persons take consecutive seats, there will be at least one vacant seat between any two
persons sitting before the first person and after the last person. Let the number of vacant seats before
the first person be x0, and the number of vacant seats between the first and the second persons be x 1,
etc., as shown in fig.
x0 x1 x2 x3 x9 x10
Clearly, the total number of vacant seats is 24-10 = 14.
 x0 + x1 + x2 + …… + x9 + x10 = 14
Where x0  0, x1  1, x2  1, x3  1, …, x9  1, x10  0.
Let x0 = y0 , = y1 + 1, x2 = y2 + 1, ….. x9 = y9 + 1, x10 = y10.
then the equation becomes
y0 + (y1 + 1) + (y2 + 1) + … + (y9 + 1) + y10 = 14
or y0 + y1 + y2 + … + y9 + y10 = 14 – 9 = 5
 Required number of ways = Number of non-negative integral solutions of the equation y 0 + y1 + y2 + …
5+11–1 15
+ y9 + y10 = 5, i.e, C11–1 = C10
But 10 persons can arrange among themselves in 10! ways.
15! 15!
Hence, the required number of ways is 15 C10  10!   10! 
10!5! 5!
Example
Find the number of non-negative integral solutions of x1 + x2 + x3 + 4x4 = 20.
Solution
20
The number of non-negative integral solutions of the given equation is equal to the coefficient of x in (1
–1 –1 –1 4 –1
– x) (1 – x) (1 – x)  (1 – x ) , which is given by
20 –3 4 –1
Coefficient of x in (1 – x) (1– x )
20 3 4 2 5 3 4 4 8
= Coefficient of x in (1 + C1x + Cx2x + C3x + 6C4x + ….) (1 + x + x + ….)
6 10 14 18 22
= 1 + C4 + C8 + C12 + C16 + C20 = 536
Example
In how many ways can 15 identical blankets be distributed among six beggers such that everyone gets at
least one blanket and two particular beggars get equal blankets and another three particular beggars get
equal blankets.
Solution
The number of ways of distributing blankets is equal to the number of solutions of the equation 3a + 2b +
c = 15 a, b, c  1, which is equal to coefficient of t in (t + t + t + t + ….)  (t + t + ….) (t + t + …..),
15 3 6 9 12 2 4 2

which is given by
Coefficient of t in (1 + t + t + t ) (1 + t + t + t + t )  (1 + t + t + …. +t ) (neglecting higher powers)
9 3 6 9 2 4 6 8 2 9
9 2 3 4 5 6 7 8 9 2 9
= Coefficient of t in (1 + t t + t + t + 2t + t + 2t + 2t ) (1 +t + t + ….. + t ) = 1 +1+1+1+1+2+1+2+2 = 12

2nd Floor, Opp. Domino’s, VIP Road, Zirakpur. Ph: 01762-351579, 9815301916, 9878170502
Chase your Dream Permutation and Combination Class : XI 54

Example
6
Find the number of integers which lie between 1 and 10 and which have the sum of the digits equal to 12.
Solution
0 1 2 9 0 1 2 9
Consider the product (x + x + x + …. + x )(x + x + x + …. + x ) …. 6 factors. The number of ways in
12
which the sum of the digits will be equal to 12 is equal to the coefficient of x in the above product. So,
required number of ways.
12 0 1 2 9 6
= Coeff. of x in (x + x + x + ….. + x )
6
12  1  x10 
= Coeff. of x in  
 1 x 
12 10 6 –6
= Coeff. of x in (1 – x ) (1 – x)
12 –6 6 10
= Coeff. of x in (1 – x) (1 – C1x + ….)
–6 –6
= Coeff. of x in (1 – x) – C1  Coeff. of x (1 – x)
12 6 2
2+6–1
C6–1 – C1 
12+6–1 6
= C6–1
= C5 – 6  C5 = 6062
17 7

Example
Find the number of ways in which two Americans, two British, one Chinese, one Dutch, and one Egyptian
can sit on a round table so that persons of the same nationality are separated.
Solution
Total number of person is 6! When A1, A2 are together,
n(A) = 5! 2! = 240
When B1, B2 together,
n(B) = 5!2! = 240
 n(A  B) = n(A) + n(B) – n(A  B)
Hence, n(A  B) = Total persons – n(A  B)
= 6! – 384
= 720 – 384 = 336
Example
There are four balls of different colors and four boxes of colors same as those of the balls. Find the
number of ways in which the balls, one in each box, could be placed in such a way that a ball does not
go to box of its own color.
Solution
Number of derangements in such problems is given by
 1 1 1 1 1
n! 1      .....  (1)n 
 1! 2! 3! 4! n! 
Hence, the required number of derangements is
1 1 1
4!      12  4  1  9
 2! 3! 4! 
Example
Find n such that n! ends in 12 zeros.
Solution
Let n! ends with e zeros
Then n  4e
Now, e = 12  n  48
They are 50, 55 etc.
Let us examine 50
 50   50   50 
We find that H5(50)! =     2    3  = 10 + 2 + 0 = 12
 5  5  5 
Which shows that 50! Ends in 12 zeros.
Also we note that none of 51, 52, 53, 54 is divisible by 5 so none of these contribute additional zero to n!.
Consequently each 50!, 51!, 52!, 53!, 54! Ends in 12 zeros.
Example
Find the number of ways in which three numbers in A.P. can be selected from the set of first n natural
numbers.
Solution
Let x, y, z be three numbers selected from the set of first n naturals such that x, y, z are in A.P.
i.e, 2y = x + z

2nd Floor, Opp. Domino’s, VIP Road, Zirakpur. Ph: 01762-351579, 9815301916, 9878170502
Chase your Dream Permutation and Combination Class : XI 55

Here we observe that 2y is always an even natural as y is a natural number and hence x + z is an even
natural number, which is possible only when either both x and z are even or both x and z are even or
both x and z are odd. Thus the number of ways in three numbers x, y, z, which are in A.P. are selected
from the set of n naturals is equivalent to the number of selection of odd and even of x and z from the
set.
Now two cases are arise –
Case 1
N is even = 2k, say
k k
In this case there are k even and k odd naturals and two numbers x, z are selected in C2 + C2 ways,
accordingly x and z are even or odd
k (k  1) n  n  n (n  2)
= 2.  .   1  ways
2 2 2  4
Case II
n is odd = 2k + 1, say
In this case these are k + 1 odd and k even natural numbers.
k+1
Two odd out (k + 1) odd natural numbers are selected in C2 ways and two even out of k even natural
k
numbers are selected in C2 ways.
Hence total number of ways of selecting x and z.
k (k  1) k (k  1) k
= k 1C2  k C2    k  1 k  1
2 2 2
2
 n  1
= k2   
 2 

2nd Floor, Opp. Domino’s, VIP Road, Zirakpur. Ph: 01762-351579, 9815301916, 9878170502
Chase your Dream Permutation and Combination Class : XI 56

1. The number of numbers of 4 digits which are not divisible by 5 are


(A) 7200 (B) 3600 (C) 14400 (D) 1800
2. A set contains (2n  1) elements. The number of subsets of the set which contain at most n elements is
n n 1 n 1 2n
(A) 2 (B) 2 (C) 2 (D) 2
3. The number of ways in which an examiner can assign 30 marks to 8 questions, awarding not less than
2 marks to any question is
(A) 21C7 (B) 30 C16 (C) 21C16 (D) None of these
4. In a certain test a i students gave wrong answers to at least i questions where i  1,2 3,.....k . No
student gave more than k wrong answers. The total numbers of wrong answers given is
(A) a1  2a2  3a3  .....  kak (B) a1  a2  a3  .....  ak
(C) Zero (D) None of these
5. Number of ways of selection of 8 letters from 24 letters of which 8 are a, 8 are b and the rest unlike is
given by
7 8 7
(A) 2 (B) 8.2 (C) 10.2 (D) None of these
6. The number of ordered triplets of positive integers which are solutions of the equation x  y  z  100
is
(A) 6005 (B) 4851 (C) 5081 (D) None of these
7. A person goes in for an examination in which there are four papers with a maximum of m marks from
each paper. The number of ways in which one can get 2m marks is
1 1
(A) 2m  3 C3 (B) (m  1)(2m 2  4m  1) (C) (m  1)(2m 2  4m  3) (D) None of these
3 3
8. The number of divisors of the form 4n  2(n  0) of the integer 240 is
(A) 4 (B) 8 (C) 10 (D) 3
9. In how many ways can Rs. 16 be divided into 4 person when none of them get less than Rs. 3
(A) 70 (B) 35 (C) 64 (D) 192
10. Two packs of 52 cards are shuffled together. The number of ways in which a man can be dealt 26 cards
so that he does not get two cards of the same suit and same denomination is
(A) 52 C26 .226 (B) 104 C26 (C) 2.52 C26 (D) None of these
11. Choose the correct number of ways in which 15 different books can be divided into five heaps of equal
number of books
15! 15!
(A) 5
(B) (C) 15 C5 (D) 15 P5
5!(3!) (3!)5
12. In a football championship, there were played 153 matches. Every team played one match with each
other. The number of teams participating in the championship is
(A) 17 (B) 18 (C) 9 (D) 13
13. Ten persons, amongst whom are A, B and C to speak at a function. The number of ways in which it can
be done if A wants to speak before B and B wants to speak before C is
10!
(A) (B) 3 ! 7 ! (C) 10 P3 .7! (D) None of these
6
14. The number of times the digit 5 will be written when listing the integers from 1 to 1000 is
(A) 271 (B) 272 (C) 300 (D) None of these

2nd Floor, Opp. Domino’s, VIP Road, Zirakpur. Ph: 01762-351579, 9815301916, 9878170502
Chase your Dream Permutation and Combination Class : XI 57

15. All possible two factors products are formed from numbers 1, 2, 3, 4....., 200. The number of factors out
of the total obtained which are multiples of 5 is
(A) 5040 (B) 7180 (C) 8150 (D) None of these
16. A car will hold 2 in the front seat and 1 in the rear seat. If among 6 persons 2 can drive, then the number
of ways in which the car can be filled is
(A) 10 (B) 20 (C) 30 (D) None of these
17. There were two women participating in a chess tournament. Every participant played two games with
the other participants. The number of games that the men played between themselves proved to
exceed by 66 the number of games that the men played with the women. The number of participants is
(A) 6 (B) 11 (C) 13 (D) None of these
18. Five balls of different colours are to be placed in three boxes of different sizes. Each box can hold all
five balls. In how many ways can we place the balls so that no box remains empty
(A) 50 (B) 100 (C) 150 (D) 200
19. A box contains two white balls, three black balls and four red balls. In how many ways can three balls
be drawn from the box if at least one black ball is to be included in the draw
(A) 64 (B) 45 (C) 46 (D) None of these
20. In how many ways can a committee be formed of 5 members from 6 men and 4 women if the committee
has at least one women
(A) 186 (B) 246 (C) 252 (D) None of these
21. Six ‘+’ and four ‘–’ signs are to placed in a straight line so that no two ‘–’ signs come together, then the
total number of ways are
(A) 15 (B) 18 (C) 35 (D) 42
22. The number of groups that can be made from 5 different green balls, 4 different blue balls and 3
different red balls, if at least 1 green and 1 blue ball is to be included
(A) 3700 (B) 3720 (C) 4340 (D) None of these
23. In an election there are 5 candidates and three vacancies. A voter can vote maximum to three
candidates, then in how many ways can he vote
(A) 125 (B) 60 (C) 10 (D) 25
24. A committee of 12 is to be formed from 9 women and 8 men in which at least 5 women have to be
included in a committee. Then the number of committees in which the women are in majority and men
are in majority are respectively
(A) 4784, 1008 (B) 2702, 3360 (C) 6062, 2702 (D) 2702, 1008
25. The number of ways in which 10 persons can go in two boats so that there may be 5 on each boat,
supposing that two particular persons will not go in the same boat is
1 1
(A) (10 C 5 ) (B) 2(8 C4 ) (C) (8 C5 ) (D) None of these
2 2
26. There are 10 persons named A, B,....J. We have the capacity to accommodate only 5. In how many
ways can we arrange them in a line if A is must and G and H must not be included in the team of 5
(A) 8 P5 (B) 7 P5 (C) 7 C3 (4!) (D) 7 C3 (5!)
27. The number of ways in which we can select three numbers from 1 to 30 so as to exclude every
selection of all even numbers is
(A) 4060 (B) 3605 (C) 455 (D) None of these
28. In a steamer there are stalls for 12 animals and there are horses, cows and calves (not less than 12
each) ready to be shipped. They can be loaded in
(A) 3  1 (C) (12)3  1
12 12
(B) 3 (D) None of these
29. There are (n  1) white and (n  1) black balls each set numbered 1 to n  1 . The number of ways in
which the balls can be arranged in a row so that the adjacent balls are of different colours is
(A) (2n  2)! (B) (2n  2)!  2 (C) (n  1)!  2 (D) 2{(n  1)!} 2
30. Sixteen men compete with one another in running, swimming and riding. How many prize lists could be
made if there were altogether 6 prizes of different values one for running, 2 for swimming and 3 for
riding
(A) 16  15  14 (B) 16  15  14
3 2 3 2
(C) 16  15  14 (D) None of these
31. The number of ways in which a committee of 6 members can be formed from 8 gentlemen and 4 ladies
so that the committee contains at least 3 ladies is
(A) 252 (B) 672 (C) 444 (D) 420

2nd Floor, Opp. Domino’s, VIP Road, Zirakpur. Ph: 01762-351579, 9815301916, 9878170502
Chase your Dream Permutation and Combination Class : XI 58

32. A student is to answer 10 out of 13 questions in an examination such that he must choose at least 4
from the first five questions. The number of choices available to him is
(A) 140 (B) 196 (C) 280 (D) 346
33. The number of ways of distributing 8 identical balls in 3 distinct boxes so that none of the boxes is
empty is
8
(A) 8 C 3 (B) 21 (C) 3 (D) 5
34. In the next World Cup of cricket there will be 12 teams, divided equally in two groups. Teams of each
group will play a match against each other. From each group 3 top teams will qualify for the next round.
In this round each team will play against others once. Four top teams of this round will qualify for the
semifinal round, where each team will play against the others once. Two top teams of this round will go
to the final round, where they will play the best of three matches. The minimum number of matches in
the next World Cup will be
(A) 54 (B) 53 (C) 38 (D) None of these
35. Let a  iˆ  jˆ  kˆ and r be a variable vector such that r .iˆ, r . jˆ and r .kˆ are positive integers. If r .a  12
then the number of values of r is
(A) 12 C9  1 (B) 12 C3 (C) 12 C9 (D) None of these
36. A man has 7 relatives, 4 women and 3 men. His wife also has 7 relatives, 3 women and 4 men. In how
many ways can they invite 3 women and 3 men so that 3 of them are the man’s relatives and 3 his
wife's
(A) 485 (B) 484 (C) 468 (D) None of these
37. A person wishes to make up as many different parties as he can out of his 20 friends such that each
party consists of the same number of persons. The number of friends he should invite at a time is
(A) 5 (B) 10 (C) 8 (D) None of these
38. The sides AB, BC, CA of a triangle ABC have respectively 3, 4 and 5 points lying on them. The number
of triangles that can be constructed using these points as vertices is
(A) 205 (B) 220 (C) 210 (D) None of these
39. Six ‘’s have to be placed in the square of the figure such that each row contains at least one . In how
many different ways can this be done

(A) 28 (B) 27 (C) 26 (D) None of these


40. The straight lines I1,I 2 ,I 3 are parallel and lie in the same plane. A total number of m points are taken on
I1, n points on I 2 , k points on I 3 . The maximum number of triangles formed with vertices at these
points are
(A) m  n  k C3 (B) m  n  k C3  mC3 nC3  k C3
(C) C3  nC3  k C3
m
(D) None of these
41. Six points in a plane be joined in all possible ways by indefinite straight lines, and if no two of them be
coincident or parallel, and no three pass through the same point (with the exception of the original 6
points). The number of distinct points of intersection is equal to
(A) 105 (B) 45 (C) 51 (D) None of these
42. There are m points on a straight line AB and n points on another line AC, none of them being the point
A. Triangles are formed from these points as vertices when (i) A is excluded (ii) A is included. Then the
ratio of the number of triangles in these two cases is
m n 2 m n 2 m n 2
(A) (B) (C) (D) None of these
m n 2 m n 2
43. How many numbers lying between 10 and 1000 can be formed from the digits 1, 2, 3, 4, 5, 6, 7, 8, 9
(repetition is allowed)
(A) 1024 (B) 810 (C) 2346 (D) None of these

44. Ten different letters of an alphabet are given. Words with five letters are formed from these given letters.
Then the number of words which have at least one letter repeated is
(A) 69760 (B) 30240 (C) 99748 (D) None of these

2nd Floor, Opp. Domino’s, VIP Road, Zirakpur. Ph: 01762-351579, 9815301916, 9878170502
Chase your Dream Permutation and Combination Class : XI 59

45. Six identical coins are arranged in a row. The number of ways in which the number of tails is equal to
the number of heads is
(A) 20 (B) 9 (C) 120 (D) 40

46. The total number of permutations of n( 1) different things taken not more than r at a time, when each
thing may be repeated any number of times is
n (n n  1) nr  1 n(n r  1)
(A) (B) (C) (D) None of these
n 1 n 1 n 1

47. How many number less than 10000 can be made with the eight digits 1, 2, 3, 4, 5, 6, 7, 0 (digits may
repeat)
(A) 256 (B) 4095 (C) 4096 (D) 4680

48. The total number of natural numbers of six digits that can be made with digits 1, 2, 3, 4, if the all digits
are to appear in the same number at least once, is
(A) 1560 (B) 840 (C) 1080 (D) 480

49. A library has a copies of one book, b copies of each of two books, c copies of each of three books and
single copies of d books. The total number of ways in which these books can be distributed is
(a  b  c  d )! (a  2b  3c  d )! (a  2b  3c  d )!
(A) (B) 2 3
(C) (D) None of these
a ! b !c ! a !(b !) (c !) a !b !c !
50. The number of ways of arranging 2m white counters and 2n red counters in a straight line so that the
arrangement is symmetrical with respect to a central mark
(m  n )! 2(m  n )!
(A) (m  n)! (B) (C) (D) None of these
m!n ! m !n !
51. Total number of four digit odd numbers that can be formed using 0, 1, 2, 3, 5, 7 are
(A) 216 (B) 375 (C) 400 (D) 720
52. The number of 4 digit numbers that can be made with the digits 1, 2, 3, 4 and 5 in which at least two
digits are identical, is
(A) 4  5!
5
(B) 505 (C) 600 (D) None of these
53. There are n straight lines in a plane, no two of which are parallel and no three pass through the same
point. Their points of intersection are joined. Then the number of fresh lines thus obtained is
n (n  1)(n  2) n (n  1)(n  2)(n  3) n(n  1)(n  2)(n  3)
(A) (B) (C) (D) None of these
8 6 8
54. How many numbers greater 40000 can be formed from the digits 2, 4, 5, 5, 7
(A) 12 (B) 24 (C) 36 (D) 48
55. In how many ways n books can be arranged in a row so that two specified books are not together
(A) n ! (n  2)! (B) (n  1)!(n  2) (C) n ! 2(n  1) (D) (n  2) n !
56. How many numbers between 5000 and 10,000 can be formed using the digits 1, 2, 3, 4, 5, 6, 7, 8, 9
each digit appearing not more than once in each number
(A) 5 8P3 (B) 5 8C3 (C) 5!  8P3 (D) 5! 8C3
57. Find the total number of 9 digit numbers which have all the digits different
(A) 9 × 9 ! (B) 9 ! (C) 10 ! (D) None of these
58. Four dice (six faced) are rolled. The number of possible outcomes in which at least one die shows 2 is
(A) 1296 (B) 625 (C) 671 (D) None of these
59. How many numbers, lying between 99 and 1000 be made from the digits 2, 3, 7, 0, 8, 6 when the digits
occur only once in each number
(A) 100 (B) 90 (C) 120 (D) 80
60. The sum of the digits in the unit place of all numbers formed with the help of 3, 4, 5, 6 taken all at a time is
(A) 18 (B) 432 (C) 108 (D) 144
61. All letters of the word AGAIN are permuted in all possible ways and the words so formed (with or
th
without meaning) are written as in dictionary, then the 50 word is
(A) NAAGI (B) IAANG (C) NAAIG (D) INAGA

2nd Floor, Opp. Domino’s, VIP Road, Zirakpur. Ph: 01762-351579, 9815301916, 9878170502
Chase your Dream Permutation and Combination Class : XI 60

62. Eight chairs are numbered 1 to 8. Two women and three men wish to occupy one chair each. First the
women choose the chairs from amongst the chairs marked 1 to 4 and then men select the chairs from
amongst the remaining. The number of possible arrangements is
(A) 6 C3  4C2 (B) 4 C2  4P3 (C) 4 P2  4P3 (D) None of these
63. If a denotes the number of permutations of x  2 things taken all at a time, b the number of
permutations of x things taken 11 at a time and c the number of permutations of x  11 things taken all
at a time such that a  182bc , then the value of x is
(A) 15 (B) 12 (C) 10 (D) 18
64. The number of ways in which ten candidates A1, A2 ,......, A10 can be ranked such that A 1 is always
above A10 is
1
(A) 5 ! (B) 2 (5 !) (C) 10 ! (D) (10!)
2
65. A dictionary is printed consisting of 7 lettered words only that can be made with a letter of the word
CRICKET. If the words are printed at the alphabetical order, as in an ordinary dictionary, the number of
word before the word CRICKET is
(A) 530 (B) 480 (C) 531 (D) 481
66. Seven different lecturers are to deliver lectures in seven periods of a class on a particular day. A, B and
C are three of the lecturers. The number of ways in which a routine for the day can be made such that A
delivers his lecture before B, and B before C, is
(A) 420 (B) 120 (C) 210 (D) None of these
67. Let A  {x : x is a prime number and x  30} . The number of different rational numbers whose
numerator and denominator belong to A is
(A) 90 (B) 180 (C) 91 (D) None of these
68. The number of numbers of 9 different non-zero digits such that all the digits in the first four places are
less than the digit in the middle and all the digits in the last four places are greater than that in the
middle is
(A) 2 (4 !) (B) (4!)2 (C) 8 ! (D) None of these
69. How many ways are there to arrange the letters in the word GARDEN with the vowels in alphabetical
order
(A) 480 (B) 240 (C) 360 (D) 120
70. A parallelogram is cut by two sets of m lines parallel to its sides. The number of parallelograms thus
formed is
(A) (m C2 )2 (B) (m 1C2 )2 (C) (m 2 C2 )2 (D) None of these
71. In a plane there are 37 straight lines of which 13 pass through the point A and 11 pass through the point
B. Besides no three lines pass through one point, no line passes through both points A and B and no
two are parallel. Then the number of intersection points the lines have is equal to
(A) 535 (B) 601 (C) 728 (D) None of these
72. There are n points in a plane of which p points are collinear. How many lines can be formed from these
points
(A) (n  p ) C2 (B) n C2  pC2 (C) n C2  pC2  1 (D) n C2  pC2  1
73. ABCD is a convex quadrilateral. 3, 4, 5 and 6 points are marked on the sides AB, BC, CD and DA
respectively. The number of triangles with vertices on different sides is
(A) 270 (B) 220 (C) 282 (D) 342
74. The number of triangles that can be formed joining the angular points of decagon, is
(A) 30 (B) 45 (C) 90 (D) 120
75. The number of triangles whose vertices are at the vertices of an octagon but none of whose sides
happen to come from the sides of the octagon is
(A) 24 (B) 52 (C) 48 (D) 16
76. In a polygon no three diagonals are concurrent. If the total number of points of intersection of diagonals
interior to the polygon be 70, then the number of diagonals of the polygon is
(A) 20 (B) 28 (C) 8 (D) None of these

2nd Floor, Opp. Domino’s, VIP Road, Zirakpur. Ph: 01762-351579, 9815301916, 9878170502
Chase your Dream Permutation and Combination Class : XI 61

77. There are n( 2) points in each of two parallel lines. Every point on one line is joined to every point on
the other line by a line segment drawn within the lines. The number of points (between the lines) in
which these segments intersect is
(A) 2n C2  2.nC1  2 (B) 2n C2  2.n C2 (C) n C2  nC2 (D) None of these
78. m parallel lines in a plane are intersected by a family of n parallel lines. The total number of
parallelograms so formed is
(m  1)(n  1) mn m (m  1) n(n  1) mn (m  1)(n  1)
(A) (B) (C) (D)
4 4 2 4
79. There are three coplanar parallel lines. If any p points are taken on each of the lines, the maximum
number of triangles with vertices at these points
(A) 3p 2 (p  1)  1 (B) 3p 2 (p  1) (C) p 2 (4p  3) (D) None of these
80. In how many ways can 15 members of a council sit along a circular table, when the Secretary is to sit on
one side of the Chairman and the Deputy secretary on the other side
(A) 2  12! (B) 24 (C) 2 × 15 ! (D) None of these
81. 20 persons are invited for a party. In how many different ways can they and the host be seated at a
circular table, if the two particular persons are to be seated on either side of the host
(A) 20! (B) 2 . 18 ! (C) 18 ! (D) None of these
82. 12 persons are to be arranged to a round table. If two particular persons among them are not to be side
by side, the total number of arrangements is
(A) 9(10 !) (B) 2 (10 !) (C) 45 (8 !) (D) 10 !
83. The number of ways that 8 beads of different colours be string as a necklace is
(A) 2520 (B) 2880 (C) 5040 (D) 4320
84. The number of ways in which 6 men and 5 women can dine at a round table if no two women are to sit
together is given by
(A) 6 ! × 5 ! (B) 30 (C) 5 ! × 4 ! (D) 7 ! × 5 !
85. In how many ways can 10 persons sit, when 6 persons sit on one round table and 4 sit on the other
round table
10 10
(A) 5 ! × 3 ! (B) 10 × 5! × 3 ! (C) C6 × 5 !× 3! (D) C6 × 5! × 3! × 2!
86. There are 20 persons among whom two are brothers . The number of ways in which we can arrange
them round a circle so that there is exactly one person between the two brothers , is
(A) 18 ! (B) 2 (18!) (C) 2 (19 !) (D) None of these
87. A family has 8 members. Four members take food two times a day on two identical round tables. For
how many months 8 members can take food by sitting in different orders (1 month = 30 days)
21
(A) 42 months (B) 21 months (C) months (D) None of these
2
88. The exponent of 3 in 100 ! is
(A) 33 (B) 44 (C) 48 (D) 52
89. The number of positive integral solutions of abc  30 is
(A) 30 (B) 27 (C) 8 (D) None of these
90. The number of 4 digit even numbers that can be formed using 0, 1, 2, 3, 4, 5, 6 without repetition is
(A) 120 (B) 300 (C) 420 (D) 20
91. The number of five digits numbers that can be formed without any restriction is
(A) 990000 (B) 100000 (C) 90000 (D) None of these
92. How many numbers less than 1000 can be made from the digits 1, 2, 3, 4, 5, 6 (repetition is not
allowed)
(A) 156 (B) 160 (C) 150 (D) None of these
93. How many even numbers of 3 different digits can be formed from the digits 1, 2, 3, 4, 5, 6, 7, 8, 9
(repetition is not allowed)
(A) 224 (B) 280 (C) 324 (D) None of these
94. A five digit number divisible by 3 has to formed using the numerals 0, 1, 2, 3, 4 and 5 without repetition.
The total number of ways in which this can be done is
(A) 216 (B) 240 (C) 600 (D) 3125

2nd Floor, Opp. Domino’s, VIP Road, Zirakpur. Ph: 01762-351579, 9815301916, 9878170502
Chase your Dream Permutation and Combination Class : XI 62

95. In a circus there are ten cages for accommodating ten animals. Out of these four cages are so small
that five out of 10 animals cannot enter into them. In how many ways will it be possible to accommodate
ten animals in these ten cages
(A) 66400 (B) 86400 (C) 96400 (D) None of these
96. How many numbers can be made with the digits 3, 4, 5, 6, 7, 8 lying between 3000 and 4000 which are
divisible by 5 while repetition of any digit is not allowed in any number
(A) 60 (B) 12 (C) 120 (D) 24
97. All possible four digit numbers are formed using the digits 0, 1, 2, 3 so that no number has repeated
digits. The number of even numbers among them is
(A) 9 (B) 18 (C) 10 (D) None of these
98. The total number of seven digit numbers the sum of whose digits is even is
(A) 9000000 (B) 4500000 (C) 8100000 (D) None of these
99. The sum of all 4 digit numbers that can be formed by using the digits 2, 4, 6, 8 (repetition of digits not
allowed) is
(A) 133320 (B) 533280 (C) 53328 (D) None of these
100. How many numbers greater than 24000 can be formed by using digits 1, 2, 3, 4, 5 when no digit is
repeated
(A) 36 (B) 60 (C) 84 (D) 120
101. How many numbers greater than hundred and divisible by 5 can be made from the digits 3, 4, 5, 6, if no
digit is repeated
(A) 6 (B) 12 (C) 24 (D) 30
102. The sum of all numbers greater than 1000 formed by using the digits 1, 3, 5, 7 no digit is repeated in
any number is
(A) 106656 (B) 101276 (C) 117312 (D) 811273
103. 3 copies each of 4 different books are available. The number of ways in which these can be arranged
on the shelf is
12! 12!
(A) 12 ! (B) (C) (D) 369,000
3 !4 ! (3 ! ) 4
104. Eleven books consisting of 5 Mathematics, 4 Physics and 2 Chemistry are placed on a shelf. The
number of possible ways of arranging them on the assumption that the books of the same subject are
all together is
(A) 4 ! 2! (B) 11! (C) 5! 4! 3! 2! (D) None of these
105. The number of positive integers which can be formed by using any number of digits from 0, 1, 2, 3, 4, 5
but using each digit not more than once in each number is
(A) 1200 (B) 1500 (C) 1600 (D) 1630
106. Let A be a set of n( 3) distinct elements. The number of triplets (x, y, z) of the elements of A in which
at least two coordinates are equal is
(B) n 3 n P3 (C) 3n  2n (D) 3n 2 (n  1)
2
(A) n P3
p
107. The number of distinct rational numbers x such that 0  x  1 and x  , where p,q  {1,2,3,4,5,6} is
q
(A) 15 (B) 13 (C) 12 (D) 11
108. The total number of 5 digit numbers of different digits in which the digit in the middle is the largest is
9
(A) 
n 4
n
P4 (B) 33 (3!) (C) 30 (3 !) (D) None of these

109. Two teams are to play a series of 5 matches between them. A match ends in a win or loss or draw for a
team. A number of people forecast the result of each match and no two people make the same forecast
for the series of matches. The smallest group of people in which one person forecasts correctly for all
matches will contain n people, where n is
(A) 81 (B) 243 (C) 486 (D) None of these
110. How many numbers greater than 5000 can be formed with the digit s 4,5,6,7 and 8 if no digit being
repeated ?
(A) 96 (B) 256 (C) 218 (D) 126

2nd Floor, Opp. Domino’s, VIP Road, Zirakpur. Ph: 01762-351579, 9815301916, 9878170502
Chase your Dream Permutation and Combination Class : XI 63

111. If the maximum number of trials required to open all locks when there are n locks and n keys is 105,
then n
(A) 13 (B) 14 (C) 35 (D) 20
112. How many numbers greater than a million can be formed with the digits 5,5,2,2,1,7,6?
(A) 1320 (B) 1180 (C) 1000 (D) 1260
113. How many different signals can be given using any number of flags from 5 flags of different colours?
(A) 325 (B) 2030 (C) 1888 (D) 1920
114. If the letters of the word SACHIN are arranged in all possible ways and these words are written out as in
dictionary, then the word SACHIN appears at serial number
(A) 602 (B) 603 (C) 600 (D) 601
115. In a test paper there are 10 questions. Number of ways in which 6 questions to be answered is
(A) 105 (B) 210 (C) 310 (D) 220
116. In how many ways 6 students and 4 teachers be arranged in a row so that no two teachers are
together?
(A) 604800 (B) 246800 (C) 258600 (D) 55500
117. Eight chairs are numbered 1 to 8. Two women and three men wish to occupy one chair each. First the
women choose the chairs from amongst he chairs marked 1 to 4; and then the men select the chairs
from amongst the remaining. The number of possible arrangements is
(A) C3  C2 (B) P2  P3
6 4 4 6 4 4 4 4
(C) C2 + C3 (D) P2 + P3
118. Harsha invites 13 guests to a dinner and places 8 of them at one table and remaining 5 at the other, the
table being round. The number of ways he can arrange the guests is
11! 13! 12!
(A) (B) 9! (C) (D)
40 40 40
119. Number of different straight line that can be formed by joining 12 different points on a plane of which 4
are collinear is
(A) 16 (B) 61 (C) 65 (D) 37
120. There are four balls of different colours and four boxes of colours, same as those of balls. The number
of ways in which the balls, one each in one box, could be placed such that any ball does not go to the
box of its own colour, is
(A) 9 (B) 13 (C) 8 (D) 16
121. At an election, a voter may vote for any number of candidates not greater than the number to be
elected. There are 10 candidates and 4 are to be elected. If a voter votes for at least one candidate,
then the number of ways in which he can vote is
(A) 6210 (B) 385 (C) 1110 (D) 5040
122. The total number of ways in which four boys and four girls can be seated around a round table, so that
no two girls sit together is equal to
(A) 7! (B) 3!4! (C) 4! (D) 4!4!
123. There are 15 couples taking part in a single tournament, the number of ways in which they can be
paired such that no two real live couples play in the same team is
 1 1 1 1 
(A) 15!  1    ..... 
15! 
(B) 15!
 1! 2! 3!
30 30 15
(C) C2 (D) C2 – C1

2nd Floor, Opp. Domino’s, VIP Road, Zirakpur. Ph: 01762-351579, 9815301916, 9878170502
Chase your Dream Permutation and Combination Class : XI 64

ANSWER KEY (JEE Mains)


1 2 3 4 5 6 7 8 9 10 11 12 13 14 15
A D A B C B C A B A A B A C B
16 17 18 19 20 21 22 23 24 25 26 27 28 29 30
B C C A B C B D D B D B B D B
31 32 33 34 35 36 37 38 39 40 41 42 43 44 45
A B B B B,c A B A C B C A B A A
46 47 48 49 50 51 52 53 54 55 56 57 58 59 60
C C A B B D B C D B A A C A C
61 62 63 64 65 66 67 68 69 70 71 72 73 74 75
C D B D A D C B C C A C D D D
76 77 78 79 80 81 82 83 84 85 86 87 88 89 90
A C D C A B A A A C B B C B C
91 92 93 94 95 96 97 98 99 100 101 102 103 104 105
C A A A B B C B A C B A C C D
106 107 108 109 110 111 112 113 114 115 116 117 118 119 120
C D D B A B D A D B A B C B A
121 122 123
B B A

2nd Floor, Opp. Domino’s, VIP Road, Zirakpur. Ph: 01762-351579, 9815301916, 9878170502
Chase your Dream Permutation and Combination Class : XI 65

Single Option Correct Type Questions


1. In a room, there are 12 bulbs of the same wattage, each having a separate switch. The number of ways
to light the room with different amount of illumination is
2 12 12 2
(A) 12 – 1 (B) 2 (C) 2 – 1 (D) 12

2. The total number of flags with three horizontal strips in order, which can be formed using 2 identical red,
2 identical green, and 2 identical white strips is equal to
(A) 4! (B) 3  (4!) (C) 2  (4!) (D) none of these

3. The number of five –digit numbers that contain 7 exactly once is


3 3 4 4
(A) (41) (9 ) (B) (37) (9 ) (C) (7) (9 ) (D) (41) (9 )

4. Total number of six – digit numbers that can be formed having the property that every succeeding digit is
greater than the preceding digit is equal to
9 10 9 10
(A) C3 (B) C3 (C) p3 (D) p3
5. The total number of five – digit numbers of different digits in which the digit in the middle is the largest is
9
(A) 
n 4
n
P4 (B) 33(3!) (C) 30(3!) (D) None of these

6. The number of four –digit number that can be made with the digits 1,2,3,4, and 5 in which at least two
digits are identical is
5
(A) 4 – 5! (B) 505 (C) 600 (D) none of these
7. Total number of words that can be formed using all letters of the word BRIJESH that neither begins with I
nor ends with B is equal to
(A) 3720 (B) 4920 (C) 3600 (D) 4800
8. Total number of six-digit numbers in which all and only odd digits appear is
5 1
(A) (6!) (B) 6! (C) (6!) (D) none of these
2 2
Total number less than 3  10 and can be formed using the digits 1,2,3 is equal to
8
9.
1 1 1 1
(A) (39  4  38 ) (B) (39  3) (C) (7  38  3) (D) (39  3  38 )
2 2 2 2
10. The total number not more than 20 digits that are formed by using the digits 0,1,2,3, and 4 is
20 20 20
(A) 5 (B) 5 – 1 (C) 5 + 1 (D) none of these
11. The number of different seven – digit numbers that can be written using only the three digits 1,2, and 3
with the condition that the digit 2 occurs twice in each number is
7 5 7 5 7 2
(A) P2 2 (B) C2 2 (C) C2 5 (D) none of these
12. The number of ways of arranging m positive and n(< m + 1) negative signs in a row so that no two
negative signs are together is
m+1 n+1 m+1 n+1
(A) Pn (B) Pm (C) Cn (D) Cm
13. The sum of all the numbers of four different digits that can be made by using the digits 0,1,2, and 3 is
(A) 26664 (B) 39996 (C) 38664 (D) none of these
14. The number of five-digit telephone numbers having at least one of their digits repeated is
(A) 90000 (B) 100000 (C) 30240 (D) 69760
15. The number of ways in which 10 candidates A1, A2, ……, A10 can be ranked such that A1 is always above
A10 is
1
(A) 5! (B) 2(5!) (C) 10! (D) (10!)
2

2nd Floor, Opp. Domino’s, VIP Road, Zirakpur. Ph: 01762-351579, 9815301916, 9878170502
Chase your Dream Permutation and Combination Class : XI 66

16. Among 10 persons, A, B, C are to speak at a function. The number of ways in which it can be done if A
wants to speak before B and B wants to speak before C is
(A) 10!/24 (B) 9!/6 (C) 10!/6 (D) none of these
17. In how many ways can a team of 6 horses be selected out of a stud of 16, so that there shall always be
three out of A B C ABC, but never AA, BB or CC together
(A) 840 (B) 1260 (C) 960 (D) 720
18. The number of ways in which the letters of the word PERSON can be placed in the squares of the given
figure so that no row remains empty is
R3

R2

R1
(A) 24  6! (B) 26  6! (C) 26  7! (D) 27  6!
19. The number of words of four letters that can be formed from the letters of the word EXAMINATION is
(A) 1464 (B) 2454 (C) 1678 (D) none of these
20. A teacher takes three children from her class to a zoo at a time, but she does not take the same three
children to the zoo more than once. She finds that she went to the zoo 84 times more than a particular
child has gone to the zoo. The number of children in her class is
(A) 12 (B) 10 (C) 60 (D) none of these
21. Number of ways in which a lawn – tennis mixed double be made from seven married couples if no
husband and wife play in the same set is
(A) 240 (B) 420 (C) 720 (D) none of these
3 2 2
22. The number of even divisors of the number N = 12600 = 2 3 5 7 is
(A) 72 (B) 54 (C) 18 (D) none of these
23. A candidate is required to answer 6 out of 10 questions, which are divided into two groups, each
containing 5 questions. He is not permitted to attempt more than 4 questions from either group. The
number of different ways in which the candidate can choose 6 questions is
(A) 50 (B) 150 (C) 200 (D) 250
24. Two teams are to play a series of five matches between them. A match ends in a win, loss, or draw for a
team. A number of people forecast the result of each match and no two people make the same forecast
for the series of matches. The smallest group of people in which one person forecast correctly for all the
matches will contain n people, where n is
(A) 81 (B) 243 (C) 486 (D) none of these
25. In an election, the number of candidates is one greater than the persons to be elected. If a voter can vote
in 254 ways, the number of candidates is
(A) 7 (B) 10 (C) 8 (D) 6
26. In an examination of nine papers, a candidate has to pass in more papers than the number of papers in
which he fails in order to be successful. The number of ways in which he can be unsuccessful is
(A) 255 (B) 256 (C) 193 (D) 319
27. A student is allowed to select at most n books from a collection of (2n +1) books. If the total number of
ways in which he can select at least one book is 63, then the value of n is
(A) 2 (B) 3 (C) 4 (D) 5
28. In a group of 13 cricket players, 4 are bowlers. Find out in how many ways can they form a cricket team
of 11 players in which at least 2 bowlers are included.
(A) 55 (B) 72 (C) 78 (D) none of these
29. ABCD is a convex quadrilateral and 3,4,5 and 6 points are marked on the sides AB, BC, CD and DA,
respectively. The number of triangles with vertices on different sides is
(A) 270 (B) 220 (C) 282 (D) 342
30. The number of triangles that can be formed with 10 points as vertices, n of them being collinear, is 110.
Then n is
(A) 3 (B) 4 (C) 5 (D) 6
31. The maximum number of points of intersection of five lines and four circles is
(A) 60 (B) 72 (C) 62 (D) none of these
32. The number of integral solutions of x + y + z =0 with x  –5, y  –5, z  – 5 is
(A) 134 (B) 136 (C) 138 (D) 140

2nd Floor, Opp. Domino’s, VIP Road, Zirakpur. Ph: 01762-351579, 9815301916, 9878170502
Chase your Dream Permutation and Combination Class : XI 67

33. In how many different ways can the first 12 natural numbers be divided into three different groups such
that numbers in each group are in A.P.?
(A) 1 (B) 5 (C) 6 (D) 4
34. If n objects are arranged in a row, then the number of ways of selecting three of these objects so that no
two of them are next to each other is
n–2 n–3 n–3
(A) C3 (B) C2 (C) C3 (D) none of these
35. The number of ways in which we can distribute mn students equally among m sections is given by
(mn)! (mn)! (mn)! m
(A) (B) m
(C) (D) (mn)
n! (n!) m!n!
36. There are three copies each of four different books. The number of ways in which they can be arranged
in a shelf is
12! 12! 21! 12!
(A) 4
(B) 3
(C) 4
(D)
(3!) (4!) (3!) 4! (4!)3 3!
37. The total number of ways in which 2n persons can be divided into n couples is
2n! 2n! 2n!
(A) (B) (C) (D) none of these
n!n! (2!)n n!(2!)n
38. Let A = {x1, x2 , x3 , ….., x7) , B = {y1 , y2, y3}. The total number of functions f: A  B that are on to and
there are exactly three element x in A such that f(x) = y 2 is equal to
(A) 490 (B) 510 (C) 630 (D) none of these
39. Number of ways in which 25 identical things be distributed among five persons if each gets odd number
of things is
25 12 14 13
(A) C4 (B) C8 (C) C10 (D) C3
40. In how many ways can 17 persons depart from railway station in 2 cars and 3 autos, given that 2
particular persons depart by the same car (4 persons can sit in a car and 3 persons can sit in an auto)?
15! 16! 17! 15!
(A) (B) (C) (D)
2! 4!(3!)3 (2!)2 4!(3!)3 2! 4!(3!)3 4!(3!)3
41. The total number of ways in which three distinct numbers in A.P. can be selected from the set
{1,2,3,…..24] is equal to
(A) 66 (B) 132 (C) 198 (D) none of these
42. The total number of times, the digit 3 will be written, when the integers having less than 4 digits are listed
is equal to
(A) 300 (B) 310 (C) 302 (D) 306
43. Two packs of 52 cards are shuffled together. The number of ways in which a man can be dealt 26 cards
so that he does not get two cards of the same suit and same denomination is
52 26 104 52
(A) C26. 2 (B) C26 (C) C26 (D) None of these
44. There are (n +1) white and (n +1) black balls, each set numbered 1 to n +1. The number of ways in which
the balls can be arranged in a row so that the adjacent balls are of different colors is
(B) (2n +2)!  2 (C) (n +1)!  2
2
(A) (2n + 2)! (D) 2{(n +1)!}
45. A is a set containing n different elements. A subset P of A is chosen. The set A is reconstructed by
replacing the elements of P. A subset Q of A is again chosen. The number of ways of choosing P and Q
so that P  Q so that P  Q contains exactly two elements is
(A) C3  2 (B) C2  3
n n n n–2 n–2
(C) 3 (D) none of these
n n n
46. n is selected from the set {1,2,3,….. 10} and the number 2 + 3 + 5 is formed. Total number of ways of
selecting n so that the formed number is divisible by 4 is equal to
(A) 50 (B) 49 (C) 48 (D) none of these
47. A man has three friends. The number of ways he can invite one friend everyday for dinner on six
successive nights so that no friends is invited more than three times is
(A) 640 (B) 320 (C) 420 (D) 510
48. A train timetable must be compiled for various days of the week so that two trains twice a day depart for
three days, one train daily for two days, and three trains once a day for two days. How many different
timetables can be compiled?
(A) 140 (B) 210 (C) 133 (D) 72
49. The total number of positive integral solution of 15 < x 1 + x2 + x3  20 is equal to
(A) 685 (B) 785 (C) 1125 (D) none of these
50. Number of natural numbers not exceeding 4321 can be formed with the digits 1,2,3,4 if repetition is
allowed is

2nd Floor, Opp. Domino’s, VIP Road, Zirakpur. Ph: 01762-351579, 9815301916, 9878170502
Chase your Dream Permutation and Combination Class : XI 68

(A) 123 (B) 113 (C) 313 (D) 222


51. The number of divisors of the form 4k + 2, k  0 of the integer 240 is
(A) 4 (B) 8 (C) 10 (D) 3
52. The number of ways, in which 10 identical objects of one kind, 10 of another kind and 10 of third kind can
be divided between two persons so that each person has 15 objects, is
(A) 136 (B) 91 (C) 45 (D) 36
53. There are 6 letters and 6 directed envelopes. The number of ways in which 4 letters are rightly placed
and 2 letters are wrongly placed, is
(A) 45 (B) 30 (C) 15 (D) zero
54. Number of 5 digit numbers which have exactly three 5’s is
(A) 774 (B) 486 (C) 288 (D) 144

Multiple Options Correct Type Questions

1. A forecast is to be made of the results of five cricket matches, each of which can be a win or a draw or a
loss for Indian team. Let
p = number of forecasts with exactly 1 error
q = number of forecasts with exactly 3 errors
r = number of forecasts with all five errors
Then the correct statement(s) is/are
(A) 2q = 5r (B) 8p = q (C) 8p = 5r (D) 2(p + r) > q
2. If a seven – digit number made up of all distinct digits 8, 7,6,4,3, x, and y is divisible by 3, then
(A) maximum value of x – y is 9
(B) maximum value of x + y is 12
(C) minimum value of xy is 0
(D) minimum value of x + y is 3
3. Given that the divisors of n = 3 . 5 . 7 are of the form 4 + 1,   0. Then
p q r

(A) p + r is always even (B) p + q+ r is always odd


(C) q can be any integer (D) if p is odd then r is even
4. Number of ways in which 30 identical things are distributed among six persons is
17
(A) C5 if each gets odd number of things.
16
(B) C11 if each gets odd number of things
14
(C) C5 if each gets even number of things (excluding 0)
15
(D) C10 if each gets even number of things (excluding 0)
5. If N denotes the number of ways of selecting r objects out of n distinct objects (r  n) with unlimited
repetition but with each object included at least once in selection, then N is equal to
r–1 r–1 r–1
(A) Cr – n (B) Cn (C) Cn – 1 (D) none of these
p q r s
6. If 10! = 2 . 3 . 5 . 7 , then
(A) 2q = p
(B) pqrs = 6
(C) number of divisors of 10! Is 280
(D) number of ways of putting 10! As a product of two natural numbers is 135
2 2 2 2 2 2 2 2
7. If P = 21(21 – 1 )(21 – 2 )(21 – 3 ) …. (21 – 10 ), then P is divisible by
(A) 22! (B) 21! (C) 19! (D) 20!
8. Let n be a four – digit integer in which all the digits are different. If x is number of odd integers and y is
number of even integers, then
(A) x < y (B) x > y (C) x + y = 4500 (D) |x–y| = 54
9. The results of 11 chess matches (as win, lose or draw) are to be forecast. Out of all possible forecast, is
(A) C8  8 (B) 2  C3 (D) 8!  3!
11 3 11
(C) 1320
10. Ritu has to answer 10 out of 13 questions in an examination. The number of ways in which she can
answer if she must answer at least 3 of the first five questions is
13 5 5 8 5 8 5 8 13 5
(A) 276 (B) C10 – C3 (C) C3. C7 + C4. C6 + C5 . C5 (D) C10 – C2
11. Let N denote the number of ways in which n boys can be arranged in a line so that 3 particular boys are
separated then, then N is equal to
(n  2)!(n  3)!
(C) n 2 C3 3!(n  3)!
n–2
(A) P3(n –3)! (B) (D) (n–3)(n –4)(n –2)!
n  5!

2nd Floor, Opp. Domino’s, VIP Road, Zirakpur. Ph: 01762-351579, 9815301916, 9878170502
Chase your Dream Permutation and Combination Class : XI 69

12. There are 25 students in a class. The teacher takes 5 of them at a time, to zoological garden as often as
he can, without taking the same 5 students more than once. Then the number of visits, the teacher
makes to the garden exceeds that of a student by
25 24 25 24 25 24
(A) C5 – C5 (B) C4 (C) C5 (D) C5 – C4
13. If N is the number of positive integral solutions of x1x2x3x4 = 770, then
(A) N is divisible by 4 distinct primes (B) N is perfect square
th
(C) N is a perfect fourth power (D) N is a perfect 8 power
14. m parallel lines in a plane are intersected by family of n parallel lines. The total number of parallelogram
so formed is
m n m(m  1)(n  1)
(A) C2. C2 (B) (C) 150, for m = 5, n = 6 (D) 25, for m = 5, n = 6
4
15. There are n straight lines in a plane, n two of which are parallel and no three pass through the same
point. Their point of intersection are joined. Then the number of fresh lines thus obtained is
(n  2)(n  3) n(n  1)(n  2)(n  3) n(n  1)(n  2)(n  3) n(n  1)(n  2)
(A) n C2 . (B) (C) (D)
4 6 8 8
Paragraph Based Questions
For Problems 1-3
Twelve persons are to be arranged around two round tables such that one table can accommodate seven
persons and another five persons only. Answer the following questions.
1. The number of ways in which these 12 persons can be arranged is
12 12
(A) C5 6!4! (B) 6!4! (C) C5 6!4! (D) none of these
2. The number of ways of arrangements if two particular persons A and B do not want to be on the same
table is
10 10 11
(A) C4 6!4! (B) 2 C6 6!4! (C) C6 6! 4! (D) none of these
3. The number of ways of arrangement if two particular persons A and B want to be together and
consecutive is
10 10 10 10
(A) C7 6! 3!2! + C5 4!5!2! (B) C5 6!3! + C7 4!5!
10 10
(C) C7 6!2! + C5 5!2! (D) none of these

For Problems 4-6


Five balls are to be placed in three boxes. Each box should hold all the five balls so that no box remains
empty.
4. The number of ways if balls are different but boxes are identical is
(A) 30 (B) 25 (C) 21 (D) 35
5. The number of ways if balls and boxes are identical is
(A) 3 (B) 1 (C) 2 (D) 35
6. The number of ways if balls as well as boxes are identical but boxes are kept in a row is
(A) 10 (B) 15 (C) 20 (D) 6

For Problems 7 – 9
There are m seats in the first row of a theatre, of which n are to be occupied .
7. The number of ways of arranging n persons if no two persons if no two persons sit side by side is
(m  n  1)! (m  n  1)! (m  n  1)! (m  n  2)!
(A) (B) (C) (D)
(m  3n  1)! (m  2n)! (m  2n  1)! (m  2n  1)!
8. If n is even, the number of ways of arranging n persons if each person has exactly one neighbour is
n m–n+1 n m–n+1 n m–n+1
(A) ( Pn/2) ( Pn/2) (B) ( Pn)( Pn/2) (C) ( Pn/2) ( Pn) (D) none of these
9. The number of ways of arranging n persons, if out of any two seats located symmetrically in the middle of
the row at least one is empty is
m/2 n m/2 m/2 n m/2 n
(A) ( Cn)(2 ) – 1 (B) Pn (C) ( Pn)(2 –1) (D) ( Pn)(2 )
For Problems 10 – 12
Five balls are to be placed in three boxes. Each can hold all the five balls. In how many different ways can we
place the balls so that no box remains empty, if
Choose the correct answer:
10. Balls and boxes are all different
(A) 60 (B) 90 (C) 150 (D) 80
11. Balls are different but boxes are identical
(C) C1  C2  C2
5 4 2
(A) 15 (B) 25 (D) 30

2nd Floor, Opp. Domino’s, VIP Road, Zirakpur. Ph: 01762-351579, 9815301916, 9878170502
Chase your Dream Permutation and Combination Class : XI 70

12. Balls are well as boxes are identical


5
(A) C3 (B) 2 (C) 1 (D) 6

Matrix Match Type Questions


Each Questions contains statements given in two columns, which have to be matched. Statements a, b, c, d
in column I have to be match with statements p, q,r,s in column II. If the correct match are a  p,  s, b  q,
b  r, c  p, c  q and d  s, then the correctly bubbled 4  4 matrix should be as follows:

1. Match the columns


Column – I Column – II
(A) Number of straight lines joining any two of 10 points (p) 30
of which four points are collinear
(B) Maximum number of points of intersection of 10 (q) 60
straight lines in the plane
(C) Maximum number of points of intersection of six (r) 40
circles in the plane
(D) Maximum number of points of intersection of six (s) 45
parabolas

2. Consider the convex polygon, which has 35 diagonals. Then match the following column.
Column – I Column – II
(A) Number of triangles joining the vertices of the (p) 210
polygon
(B) Number of points of intersections of diagonal which (q) 120
lies inside the polygon
(C) Number of triangles in which exactly one side is (r) 10
common with that of polygon
(D) Number of triangles in which exactly two sides are (s) 60
common with that of polygon

3. A function is defined as f : {a1 , a2, a3, a4, a5, a6}  {b1, b2, b3}
Column – I Column – II
(A) Total number of function f: {1,2,3, 4,5}  {1,2,3,4,5} (p) Divisible by 11
that are on to and f(i)  i is equal to
If x1x2x3 = 2  5  7 , then the number of solution set
2
(B) (q) Divisible by 7
for (x1,x2,x3) where xi  > 1 is
(C) Number of factors of 3780 are divisible by either 3 or (r) Divisible by 3
2 or both is
Total number of divisors of n = 2  3  5 that are
5 4 10
(D) (s) Divisible by 4
of the form 4 + 2,   1 is

4. Match the columns


Column – I Column – II
(A) Four dice (six faced) are rolled. The number of (p) 210
possible outcomes in which at least one dice show 2
is
(B) Let A be the set of 4-digit number a1a2a3a4, where (q) 480
a1 > a3 > a4. Then n(A) is equal to
(C) The total number of three-digit numbers, the sum of (r) 671
whose digits is even ,is equal to
(D) The number of four digit numbers that can be formed (s) 450
from the digits 0, 1,2,3,4,5,6,7 so that each number
contain digit 1 is

2nd Floor, Opp. Domino’s, VIP Road, Zirakpur. Ph: 01762-351579, 9815301916, 9878170502
Chase your Dream Permutation and Combination Class : XI 71

5. Match the columns


Column – I Column – II
(A) The number of five – digit numbers having the (p) > 70
product of digits 20 is
(B) A closest has five pairs of shoes. The number of (q) < 60
ways in which four shoes can be drawn from it such
that there will be no complete pair is
(C) Three ladies have each brought their one child for (r)  (50, 110)
admission to a school. The principal wants to
interview the six persons one by one subject to the
condition that no mother is interviewed before her
child. The number of ways in which interviews can be
arranged is
(D) The figures 4,5,6,7,8 written in every possible order. (s) (40, 70)
The number of numbers greater than 56000 is

6. Match the column- I with column – II


Column – I Column – II
(A) If a denotes the number of permutations of x + 2 (p) 6
things taken all at a time, b the number of
permutations of x things taken 11 at a time and c the
number of permutations of x –11 things taken all at a
time such that a = 182bc, then the value of x is
product of
(B) The number of six – digit numbers that can be made (q) 5
with the digits 0,1,2,3,4 and 5 so that even digits
occupy odd places is a product of
(C) The number of five – digit numbers that can be made (r) 4
using the digits 1 and 2 and in which at least one
digit is different is a product of
(D) In a polygon the number of diagonals is 54. The (s) 3
number of sides of the polygon is a product of

7. How many seven letters words can be formed by using the letters of the word SUCCESS so that
Column – I Column – II
(A) The two C are together but not two S are together (p) 60
(B) Neither two C nor two S are together (q) 96
(C) All C are together and S are together and E always comes before (r) 24
U
(D) U is the starting letter of the word (s) 12
8. Match the following
Column – I Column – II
(A) There are 8 Hindi novels and 6 English novels. 4 Hindi novels and (p) 5.(8!)
3 English novels are selected and arranged in a row such that they 56
are alternate then no. of ways is
(B) Number of arrangements of letters of the word CONCRETE such (q) 5(8!)
that no two vowels are together is
(C) There are 10 AC in a hall. In how many ways they can be operated (r) 848
such that atleast 4 AC is on?
(D) A = {1,2,3,…..,10,11}. Number of subsets of A having atleast 6 (s) 1024
elements is equal to
8 6
(t) P4 . P3

2nd Floor, Opp. Domino’s, VIP Road, Zirakpur. Ph: 01762-351579, 9815301916, 9878170502
Chase your Dream Permutation and Combination Class : XI 72

9. Match the entries of Column – I with those of Column – II


Column – I Column – II
10 10
(A) The value of x satisfying the inequality Cx–1 > 2. Cx (p) 9
is
(B) A person wishes to make up as many different parties as he can (q) 7
out of his 20 friends such that each party consists of the same
number of persons. The number of friends he should invite at a
time is less than or equal to
(C) There are four letters and four directed envelopes. The number of (r) 8
ways in which all the letters can be put in the wrong envelopes is
(D) If a denotes the number of permutations of x +2 things taken all at (s) 10
a time , b the number of permutations of x things taken 11 at a time
and c the number of permutations of x – 11 things taken all a time
such that a = 182 bc, then the value of x is greater than or equal to
(t) 12

Integer Type Questions


1. There are n distinct white and n distinct black balls. If the number of ways of arranging them in a row so
that neighbouring balls are of different colors is 1152, then value of n is _____.
2. Numbers from 1 to 1000 are divisible by 60 but not by 24 is
3. A class has three teachers, Mr. P, Ms. Q, and Mrs. R and six student A, B, C, D, E, F. Number of ways in
which they can be seated in a line of 9 chairs, if between any two teachers there are exactly two
students, is k!(18), then the value of k is _____.
4. 10. Number of permutations of 1,2,3,4,5,6,7,8 and 9 taken all at a time are such that the digit
1 appearing somewhere to the left of 2
3 appearing to the left of 4 and 5 somewhere to the left of 6, is k  7! Then the value of k is _____.
5. There are 2 women participating in a chess tournament. Every participant played 2 games with the other
participants. The number of games that the men played between themselves exceeded by 66 as
compared to the number of games that the men played with the women. If the number of participants is
n, then the value of n – 6 is ___.
6. The number of three – digit numbers having only two consecutive digits identical is N, then the value of
1/2
(N/2) is ___.
7. Number of ways in which 5 A’s and 6 Bs can be arranged in a row which reads the same backwards and
forwards is N, then the value of N/2 is _____.
8. If N is the number of ways in which a person can walk up a stairway which has 7 steps if he can take 1 or
2 steps up the stairs at a time, then the value of N/3 is ____.
9. A man has 3 friends. If N is number of ways he can invite one friend everyday for dinner on 6 successive
nights o that no friends is invited more than 3 times then the value of N/170 is ____.
10. An eight digit number is formed from 1,2,3,4 such that product of all digits is always 3072, the total
8
number of ways is (23. Ck), where the value of k is ____.
11. There are red, green and white identical balls, each being 10 in number. The number of selections of 10
balls in which the number of red balls is double the number of green ball is ___.
12. The number of ways can 14 identical toys distributed among three boys so that each one gets atleast
n
one toy and no two boys get equal number of toys is n, then is equal to ___
10
13. The number of ordered triplets of positive integers which satisfy the inequality 20  x + y + z  50 is p,
p  631
then is equal to _____
2000
14. The number of 6 digit numbers that contains 6 exactly once is 7k1.3k2 , then |k2 – k1| is
equal to ___
15. The symbols +,+,,, , , are placed in the squares of the adjoining figure. The number
of ways of placing symbols so that no row remains empty is k, then the ten’s digit of k is
____
16. The total number of non-similar triangle which can be formed such that all the angles of the
n
triangle are integers in n, then is equal to ____
900

2nd Floor, Opp. Domino’s, VIP Road, Zirakpur. Ph: 01762-351579, 9815301916, 9878170502
Chase your Dream Permutation and Combination Class : XI 73

17. If total number of integral solutions of xyz = 42 is n then |n – 103| is equal to


18. If the number of integral solution of x + y + z = 13 such that x  1, y > 2 and is z  3 is m, then the
number of divisors of m is
19. A six faced ordinary cubical die marked with alphabets A, B, C ,D ,E, F is thrown ‘n’ times and the total
number of ways in which among the alphabets A, B, C, D , E and F only three of them appear in the list is
10800. Then n is equal to
20. The number of seven digit numbers in which every digit is either greater than or equal to immediately
preceeding one is n, then the unit’s digit of n is

Answer key
Single answer correct
1 2 3 4 5 6 7 8 9 10 11 12 13 14 15
C A A A D B A A C A B C C D D
16 17 18 19 20 21 22 23 24 25 26 27 28 29 30
C C B B B B B C B C B B C D C
31 32 33 34 35 36 37 38 39 40 41 42 43 44 45
C B D A B A C A C D B A A D B
46 47 48 49 50 51 52 53 54
B D B A C A B C A
More than One Correct
1 2 3 4 5 6 7 8 9 10 11 12 13
A,B, ALL A,C A,C A,C B,C,D B,C,D A,D A,B, ALL ALL C,D B,C,D
D C
14 15
A,C A,C
Comprehension
1 2 3 4 5 6 7 8 9 10 11 12
C B A A D A C A D C B B
(Matrix Match)
1 2
A  r; b  s; c  p; d  q A  q; b  p; c  s; d  r
3 4

A  p,s; b  q,r; c  p,s; d  r A  r; B  p; C  s; D  q


5 6
A  q,s; b  p,r; c  p,r; d  p,r A  r,s; b  p,r; c  p,q; d  r,s
7 8
A  (r), (B)  (q), (C)  (s), (D)  (p) A  (q,t), (B)  (p), (C)  (r), (D)  (s)
9
A  (p,r,s), (B)  (s,t), (C)  (p), (D)  (p,q,r,s,t)
Integer Type Questions
1 2 3 4 5 6 7 8 9 10 11 12 13 14
4 8 6 9 7 9 5 7 3 3 or 5 4 6 9 6
15 16 17 18 19 20
8 3 2 6 6 5

2nd Floor, Opp. Domino’s, VIP Road, Zirakpur. Ph: 01762-351579, 9815301916, 9878170502

You might also like